Auditing Round 2

अब Quizwiz के साथ अपने होमवर्क और परीक्षाओं को एस करें!

the scope of Smith's inquiry and analytical procedures is not restricted. AR-C 90.A2 states that a review report on the balance sheet of Cone Company may be issued only if the scope of Smith's inquiry and analytical procedures is not restricted. For example, since the balance in retained earnings includes current-year net income, inquiry and analytical procedures concerning net income cannot be restricted. However, it is not necessary to also compile and report on the related statements of income, retained earnings, and cash flows. If Smith is aware of material modifications needed for the balance sheet to conform with GAAP, his review report should so state. Whether or not Cone is a new client and the timing of the acceptance of the engagement are irrelevant to the question.

Smith, CPA, has been asked to issue a review report on the balance sheet of Cone Company, a nonpublic entity (nonissuer), and not on the other related financial statements. Smith may do so only if: Smith compiles and reports on the related statements of income, retained earnings, and cash flows. Smith is not aware of any material modifications needed for the balance sheet to conform with GAAP. the scope of Smith's inquiry and analytical procedures is not restricted. Cone is a new client, and Smith accepts the engagement after the end of Cone's fiscal year.

Five years AU-C 230.17 states that the retention period "should not be shorter than five years from the report release date."

Under the Statements on Auditing Standards (SASs), the auditor should adopt reasonable procedures to retain and access audit documentation for a period of time sufficient to meet the needs of his or her practice and to satisfy any applicable legal or regulatory requirements for records retention. Such retention period, however, should not be fewer than how many years following the report release date? One year Three years Five years Seven years

obtain a representation letter from the successor auditor.

When the auditor reissues a report of the financial statements, the responsibility of the auditor with respect to the reissued report is which of the following? The auditor: should adjust the financial statements as needed or qualify the opinion. must disclaim an opinion. should perform limited review procedures as necessary. obtain a representation letter from the successor auditor.

obtain a representation letter from the successor auditor. When the auditor reissues a report of the financial statements, the auditor should read the financial statements of the subsequent period, compare the prior-period financial statements to the subsequent period, request a written representation from management, and obtain a representation letter from the successor auditor. Review procedures are not performed. AU-C 560.19

When the auditor reissues a report of the financial statements, the responsibility of the auditor with respect to the reissued report is which of the following? The auditor: should adjust the financial statements as needed or qualify the opinion. must disclaim an opinion. should perform limited review procedures as necessary. obtain a representation letter from the successor auditor.

Design and implementation of controls The auditor should obtain an understanding of the five components of internal control sufficient to assess the risk of material misstatement of the financial statements whether due to error or fraud, and to design the nature, timing, and extent of further audit procedures. The auditor obtains this understanding by performing risk assessment procedures to evaluate the design of controls relevant to an audit of financial statements and determine whether they have been implemented. Operating effectiveness is only required to be assessed if a control reliance strategy is elected or required in the audit

Which of the following is required to be evaluated by a financial statement auditor of a nonissuer in order to assess the risk of material misstatement and design the detailed audit plan? Design and implementation of controls Implementation and operating effectiveness of controls Design and operating effectiveness of controls Design, implementation, and operating effectiveness of controls

describe the nature of the departure from an applicable financial reporting framework in the CPA's report and state the effects on the financial statements, if practicable. The key to this question is the phrase "unaudited financial statements." A CPA would disclaim an opinion on the unaudited financial information of a public entity (an issuer) when he is associated with the financial statements but has not reviewed or audited them. The disclaimer would state that the financial statements "were not audited by us and, accordingly, we do not express an opinion on them." If there should be a material departure from an applicable financial reporting framework that management refuses to correct, the CPA should modify the language in the report to describe the departure. The CPA is not expressing limited assurance in this circumstance; he is expressing no assurance when there is a disclaimer. The report cannot be restricted to the entity's management and board of directors if it is accompanying financial information required to be submitted to a third party. As an audit has not been performed, the CPA would not issue a qualified or adverse opinion.

A CPA concludes that the unaudited financial statements on which the CPA is disclaiming an opinion are not in conformity with an applicable financial reporting framework because management has failed to capitalize leases. The CPA suggests appropriate revisions to the financial statements, but management refuses to accept the CPA's suggestions. Under these circumstances, the CPA ordinarily would: express limited assurance that no other material modifications should be made to the financial statements. restrict the distribution of the CPA's report to management and the entity's board of directors. issue a qualified opinion or adverse opinion depending on the materiality of the departure from an applicable financial reporting framework. describe the nature of the departure from an applicable financial reporting framework in the CPA's report and state the effects on the financial statements, if practicable.

an adverse opinion. In the independent accountant's report for an examination of prospective financial statements, the CPA's opinion would be modified (changed from the standard, unmodified report) according to the following situations: Qualified opinion: In the accountant's opinion, the prospective financial statements depart from AICPA presentation guidelines (such as failure to disclose significant accounting policies). Adverse opinion:There is a measurement departure, an unreasonable assumption, or a limitation on the scope of the practitioner's examination that prohibits the issuance of an unmodified opinion.Departure from AICPA presentation guidelines is because the presentation fails to disclose assumptions that appear to be significant.One or more significant assumptions do not provide a reasonable basis for the forecast. Disclaimer of opinion: The accountant was precluded from applying one or more procedures considered necessary in the circumstances in order to express any opinion. There is no such thing as a "pro forma opinion." An unmodified opinion can have an emphasis-of-matter paragraph added in order to present explanatory comments or informational material; however, the circumstance stated in the question requires that an adverse opinion be issued.

A CPA is engaged to examine an entity's financial forecast. The CPA believes that several significant assumptions do not provide a reasonable basis for the forecast. Under these circumstances, the CPA should issue: an adverse opinion. a pro forma opinion. a qualified opinion. an unmodified opinion with an emphasis-of-matter paragraph.

Indicate that the accountant's conclusion is not modified with respect to this matter -The emphasis-of-matter paragraph, if appropriate, should be included within a separate section of the report with the heading "Emphasis of Matter"; immediately follow the "Basis for Opinion" paragraph; include a clear reference to the matter being emphasized and to where relevant disclosures that fully describe the matter can be found in the financial statements; and indicate that the accountant's conclusion is not modified with respect to the matter emphasized.

A discretionary emphasis-of-matter paragraph is included in a review report. How is the standard review report modified? Qualify the review report for the matter emphasized Indicate that the accountant's conclusion is not modified with respect to this matter Include the matter in the introductory paragraph of the review report Move the footnote disclosure to the review report under the heading "emphasis of matter"

ethical requirements relevant to the group audit.

A group engagement team of a nonissuer should ask a component auditor to communicate whether it complied with: contract billing requirements related to the group audit. generally accepted accounting principles relevant to the group audit. state licensure requirements applicable to all group locations. ethical requirements relevant to the group audit.

ethical requirements relevant to the group audit. Statement on Quality Control Standards (SQCS) 8, A Firm's System of Quality Control, states that the quality control policies and procedures applicable to a firm's accounting and auditing practice should encompass the following elements: leadership responsibilities for quality within the firm, relevant ethical requirements, acceptance and continuance of client relationships and specific engagements, human resources, engagement performance, and monitoring. These elements would also apply to a component auditor. AU-C 600 states that a component auditor could be part of the group audit engagement partner's firm in a different location, a network firm, or another firm. Group engagement partners are required to gain an understanding of the component auditors and determine the extent the group engagement team will be involved in the work of the component auditors

A group engagement team of a nonissuer should ask a component auditor to communicate whether it complied with: contract billing requirements related to the group audit. generally accepted accounting principles relevant to the group audit. state licensure requirements applicable to all group locations. ethical requirements relevant to the group audit.

The service organization's system calculates accounts receivable balances. AT-C 320, Reporting on an Examination of Controls at a Service Organization Relevant to User Entities' Internal Control Over Financial Reporting, contains performance and reporting requirements and application guidance for a service auditor examining controls at organizations that provide services to user entities when those controls are likely to be relevant to user entities' internal control over financial reporting. When the user entity initiates transactions and the service organization executes and does the accounting processing of those transactions, there is a high degree of interaction between the activities at the user entity and those at the service organization. Auditing and examination standards require the accountant to obtain an understanding of each of the five components of an entity's internal control sufficient to plan the engagement. This understanding may encompass controls placed in operation by the entity and by service organizations whose services are part of the entity's information system.

A service organization provides processing services for a client's sales orders. Which of the following information is relevant when gathering data for the report on the service organization's internal controls? Credit limits are established and updated by the client's credit department. The client's sales manager reviews accounts receivable balances. The client's data entry clerk used the sales manager's password to make unauthorized changes to customer prices. The service organization's system calculates accounts receivable balances.

the auditor's risk assessment procedures have not identified any effective controls relevant to the assertion AU-C 330.A4 states that "the auditor may determine that...performing only substantive procedures is appropriate for particular assertions, and therefore, the auditor excludes the effect of controls from the relevant risk assessment. This may be because the auditor's risk assessment procedures have not identified any effective controls relevant to the assertion or because testing controls would be inefficient, and therefore, the auditor does not intend to rely on the operating effectiveness of controls in determining the nature, timing, and extent of substantive procedures." If the auditor determines that effective controls exist for the relevant assertion level, the auditor might choose to perform tests of controls for operating effectiveness depending on whether such tests would be efficient or not.

After assessing the risk of material misstatement at the relevant assertion level, the auditor determines that performing only substantive procedures is appropriate for specific relevant assertions and risk. The auditor came to this conclusion because: the auditor's risk assessment procedures determined that effective controls exist for the relevant assertion. the auditor's risk assessment procedures have not identified any effective controls relevant to the assertion the auditor's materiality level was low. substantive procedures were performed for the relevant assertion level in the prior year's audit.

The accountant should include a final paragraph in the accountant's compilation report. According to AR-C 80.22, independence is impaired if, during the engagement, the accountant has a direct financial interest in the client. Ownership is a form of direct financial interest. If the accountant is not independent, the accountant should include a final paragraph in the accountant's compilation report referencing the nonindependence of the accountant.

An accountant compiled the financial statements of a nonissuer in accordance with Statements on Standards for Accounting and Review Services (SSARS). If the accountant has an ownership interest in the entity, which of the following statements is correct? The accountant should refuse the compilation engagement. A report need not be issued for a compilation of a nonissuer. The accountant should include the disclaimer "I am an owner of the entity" in the report. The accountant should include a final paragraph in the accountant's compilation report.

Make inquiries about subsequent events. "Make inquiries about subsequent events" is correct. When evidence or information surfaces of subsequent events that require adjustment of, or disclosure in, the financial statements comes to the accountant's attention, the accountant should request that management consider whether each such event is appropriately reflected in the financial statements in accordance with the applicable financial reporting framework. The remaining answer choices are incorrect because they go beyond what is expected of the accountant to perform during a review engagement. The objective of a review of financial statements is to obtain limited assurance as to whether there are material misstatements using methods of inquiry and analytical procedures. Internal control structure reviews, bank confirmations, and tests of controls are all examples of procedures typically beyond the scope and objective of a review of the financial statements. These are procedures more commonly performed during the course of an audit.

An accountant is reviewing the financial statements of a nonpublic entity in accordance with the Statements on Standards for Accounting and Review Services (SSARS). The accountant most likely would perform which of the following procedures? Obtain an understanding of the internal control structure. Make inquiries about subsequent events. Send bank account confirmations. Perform limited tests of controls.

Substantive procedures for the period between the interim date and the balance sheet date Substantive procedures are performed to detect material misstatements at the relevant assertion level, and include tests of details of classes of transactions, account balances, and disclosures and substantive analytical procedures. The auditor may perform substantive procedures at an interim date or at period-end. When substantive procedures are performed at an interim date, the auditor should perform further substantive procedures or substantive procedures combined with tests of controls to cover the remaining period that provide a reasonable basis for extending the audit conclusions from the interim date to the period-end.

An auditor decides to perform substantive tests on a client's property and equipment balance as of an interim date. The auditor has not obtained evidence about the operating effectiveness of relevant controls. What additional work must be performed to extend the audit conclusions from the interim date to the balance sheet date? Tests of controls for the period between the beginning of the fiscal year and the interim date Analytical comparison of the current-year interim balance with the prior-year interim balance Tests of controls for the period between the interim date and the balance sheet date Substantive procedures for the period between the interim date and the balance sheet date

tolerable rate (7%) was less than the achieved upper precision limit (8%). The auditor made a preliminary judgment that up to 7% of the sales invoices lacking approval would be acceptable. Thus, the tolerable deviation rate was 7%. Since the achieved upper precision limit or achieved deviation rate of sample was 8%, the auditor would either have to conclude the control procedure of credit approval was unacceptable or increase the preliminary assessment of control risk to allow for a decrease in the achieved upper precision limit to a level below the tolerable deviation rate.

An auditor desired to test credit approval on 10,000 sales invoices processed during the year. The auditor designed a statistical sample that would provide a 1% risk of assessing control risk too low (99% confidence) that not more than 7% of the sales invoices lacked approval. The auditor estimated from previous experience that about 2.5% of the sales invoices lacked approval. A sample of 200 invoices was examined and seven of them were lacking approval. The auditor then determined the achieved upper precision limit to be 8%. In the evaluation of this sample, the auditor decided to increase the level of the preliminary assessment of control risk because the: expected deviation rate (7%) was more than the percentage of errors in the sample (3.5%). expected deviation rate (2.5%) was less than the tolerable rate (7%). achieved upper precision limit (8%) was more than the percentage of errors in the sample (3.5%). tolerable rate (7%) was less than the achieved upper precision limit (8%).

the cost and effort of selecting additional sample items is low. The risk of incorrect rejection is the chance that the statistical evidence might fail to support fair statement of a correct book value. This type of error generally results in addition testing of sample items. When the cost and effort of selecting additional sample items is low, the auditor would increase the risk of incorrect rejection. That way, if the sample results failed to support the fair statements of a book value, it would not be costly or time prohibitive to select additional sample items.

An auditor may decide to increase the risk of incorrect rejection when: many differences (audit value minus recorded value) are expected. the cost and effort of selecting additional sample items is low. increased reliability from the sample is desired. initial sample results do not support the planned level of control risk.

Presentation and disclosure: No; Rights and obligations: Yes An organization follows up on errors to the monthly statements to determine the accounts receivable dollar amount that the organization has the right to receive. The presentation assertion deals with whether components of the financial statements are properly listed and disclosed. The presentation would not be affected by management's following-up on errors reported by customers.

An auditor observed that a client mails monthly statements to customers. Subsequently, the auditor reviewed evidence of follow-up on the errors reported by the customers. This test of controls most likely was performed to support management's financial statement assertion(s) of: Presentation and disclosure: Yes; Rights and obligations: Yes Presentation and disclosure: Yes; Rights and obligations: No Presentation and disclosure: No; Rights and obligations: Yes Presentation and disclosure: No; Rights and obligations: No

dates checks are deposited per bank statements with the dates remittance credits are recorded. Lapping involves the altering of accounts receivable when cash that is intended for the payment of a receivable is misappropriated. The first receivable collected is used to cover the misappropriation, while the second receivable is collected to account for the first, and so on. In order to prevent lapping, the best internal control activity would be the segregation of duties between those receiving cash and those posting to the accounts receivable ledger. To uncover the scheme, the best solution would be for the auditor to compare the dates checks are deposited with the dates of remittance credits. If lapping were to occur, over time the posting of remittance credits should occur prior to the date the check is deposited.

An auditor suspects that a client's cashier is misappropriating cash receipts for personal use by lapping customer checks received in the mail. In attempting to uncover this embezzlement scheme, the auditor most likely would compare the: dates checks are deposited per bank statements with the dates remittance credits are recorded. daily cash summaries with the sums of the cash receipts journal entries. individual bank deposit slips with the details of the monthly bank statements. dates uncollectible accounts are authorized to be written off with the dates the write-offs are actually recorded.

the needs of third parties who anticipated the original engagement and now must rely on the more limited agreed-upon procedures engagement. Third parties are not a consideration in an agreed-upon procedures engagement as such engagements are intended for the benefit of specified parties who are mentioned in the report and are responsible for the procedures performed.

Before an accountant who was engaged to perform another form of engagement agrees to change the engagement to an engagement to apply agreed-upon procedures, he or she should consider all of the following, except: the needs of third parties who anticipated the original engagement and now must rely on the more limited agreed-upon procedures engagement. the additional effort required to complete the original engagement. the reason given for the request, particularly the implications of a restriction on the scope of the original engagement or the matters to be reported. the possibility that certain procedures performed as part of another type of engagement are not appropriate for inclusion in an engagement to apply agreed-upon procedures.

critical review of the judgment exercised at every level of supervision. The planning and supervision standard is ordinarily interpreted to require critical review of the judgment exercised at every level of supervision. The first standard of fieldwork requires that the evidence gathering phase of the audit be planned and properly supervised. A system of quality controls should be in place that gives reasonable assurance that standards of quality are maintained within the firm with regard to supervision. Policies and procedures for the conduct and supervision at all organization levels should exist to provide reasonable assurance that the work performed meets the firms' standards of quality. This would require critical review of the judgment exercised at every level of supervision.

Effective policies and procedures ask that the auditor adequately plan the work and properly supervise any assistants. These policies and procedures are ordinarily interpreted to require: thorough review of the existing safeguards over access to assets and records. limited review of the indications of employee fraud and noncompliance with laws and regulations. objective review of the adequacy of the technical training and proficiency of firm personnel. critical review of the judgment exercised at every level of supervision.

Monthly statements are mailed to all customers with outstanding balances. Mailing monthly statements to all customers with outstanding balances is the internal control that, if properly designed and implemented, most likely could assist MMI in preventing or detecting the misappropriation of a customer's check. The customer provides an independent check on the customer account balance and is a source of information which is external to the organization.

Field, CPA, is auditing the financial statements of Miller Mailorder, Inc., (MMI) for the year ended January 31, 20X1. Field has compiled a list of possible errors and fraud that may result in the misstatement of MMI's financial statements, and a corresponding list of internal controls that, if properly designed and implemented, could assist MMI in preventing or detecting the errors and fraud. Select the internal control that most likely could assist MMI in preventing or detecting the misappropriation of a customer's check. Monthly statements are mailed to all customers with outstanding balances. Remittance advices are separated from the checks in the mailroom and forwarded to the accounting department. Total amounts posted to the accounts receivable ledger from remittance advices are compared with the validated bank deposit slips. An employee other than the bookkeeper periodically prepares a bank reconciliation.

Selling inventory on account The current ratio is computed by dividing current assets (CA) by current liabilities (CL). Thus, a current ratio of 4:1 means Heath has four times more current assets than current liabilities. Both inventory and accounts receivable are current assets which appear in the numerator when computing the current ratio. However, since inventory is sold on account at a price greater than its cost, current assets will increase by the sales price amount and decrease by the cost of inventory sold. Thus, selling inventory on account will increase the current ratio. Purchasing inventory on account increases current assets and current liabilities by the same amount. However, the absolute change is not all you need to consider. Because the current ratio is 4:1, the proportionate effect of an increase in CL will be greater than the effect of an equal increase in CA. Thus, the current ratio will decrease. Collecting an account receivable has no effect on the current ratio. Current assets increase (cash) and decrease (AR) by the same amount. Machinery is a noncurrent asset. Thus, purchasing machinery for cash decreases CA (cash) which decreases the current ratio.

Heath Co.'s current ratio is 4:1. Which of the following transactions would normally increase its current ratio? Purchasing inventory on account Selling inventory on account Collecting an account receivable Purchasing machinery for cash

I: Explicitly; II: Explicitly

How does an accountant make the following representations when issuing the standard report for the compilation of a nonissuer's financial statements? The financial statements have not been audited. The accountant has compiled the financial statements. I: Implicitly; II: Implicitly I: Explicitly; II: Explicitly I: Implicitly; II: Explicitly I: Explicitly; II: Implicitly

The classes of transactions in the issuer's operations that are significant to the issuer's financial statements Internal controls consist of five interrelated components: control environment, risk assessment, information and communication, control activities, and monitoring. Information and communication systems support the identification, capture, and exchange of information and consist of the procedures and records relevant to financial reporting objectives (including the accounting system). As such, the auditor should obtain sufficient knowledge of the information system to understand the following: Significant classes of transactions in the entity's operations Manual and automated procedures by which transactions are initiated, authorized, recorded, processed, and reported in the financial statements The related accounting records supporting information, and specific accounts in the financial statements involved in initiating, authorizing, recording, processing, and reporting transactions The financial reporting process used to prepare the entity's financial statements, including significant accounting estimates and disclosures Controls surrounding journal entries Assessing management's integrity, ethical values, philosophy, and operating style are related to the control environment component, not information and communication.

If an auditor is obtaining an understanding of an issuer's information and communication component of internal control, which of the following factors should the auditor assess? The integrity and ethical values of top management The philosophy and operating style of management to promote effective internal control over financial reporting The classes of transactions in the issuer's operations that are significant to the issuer's financial statements The oversight responsibility over financial reporting and internal control by the board or audit committee

Neither I nor II If adequately disclosed in the financial statements, an uncertainty about an entity's ability to continue as a going concern or other accounting matters (other than those involving a change in accounting principles) may be, at the accountant's discretion, emphasized in the accountant's report (but will not require a modification to the standard compilation or review report).

If properly disclosed in the financial statements, which of the following would ordinarily cause an accountant to modify his or her standard compilation or review report? Uncertainty about the entity's ability to continue as a going concern Inconsistency in the application of accounting principles I only II only Both I and II Neither I nor II

sales journal to the shipping documents. If the auditor is concerned that sales are overstated, the auditor is worried about the occurrence assertion regarding sales. Therefore, the auditor would want to verify that all recorded sales are supported by shipping documents, cash receipts, and/or third-party sales orders. The auditor would start with the sales journal and trace their way to the supporting documentation such as shipping documents.

If the objective of an auditor's test of details is to detect the overstatement of sales, the auditor should trace transactions from the: sales journal to the shipping documents. cash receipts journal to the customer's purchase orders. customer's purchase orders to the sales journal. shipping documents to the cash receipts journal.

Perform substantive procedures for all relevant assertions related to each material class of transactions The auditor should plan and perform substantive procedures to be responsive to the related assessment of the risk of material misstatement. The auditor's risk assessment is judgmental, however, and may not be sufficiently precise to identify all risks of material misstatement. In order to obtain sufficient and appropriate audit evidence to draw reasonable conclusions on which to base the audit, the auditor should perform substantive procedures consisting of inspection of records or documents, inspection of tangible assets, observation and inquiry, confirmation, recalculation and reperformance, and analytical procedures.

In an audit of financial statements for which an auditor's assessment of risk is judgmental and may not be sufficiently precise to identify all risks of material misstatement, the auditor should take which of the following actions? Consider whether risk assessment procedures are appropriate given preliminary levels of materiality and tolerable misstatement Determine the effectiveness of general controls over classes of transactions characterized by high transaction volume Discuss strategies to eliminate such risks with top management or those with equivalent authority and responsibility Perform substantive procedures for all relevant assertions related to each material class of transactions

an override of internal controls by a low-level employee. "An override of internal controls by a low-level employee" is the correct answer. Internal controls are designed to provide reasonable assurance the entity's objectives are achieved. These systems should be designed to ensure that "low-level" employees cannot override the control. With that said, it can be far more challenging to prevent "higher-level" employees, such as management, from inappropriately overriding a control. This is due to the special authority they hold within the organization and fact management is rarely monitored as actively as "low-level" employees. The remaining answer choices all represent inherent limitations to internal controls, namely that it is difficult if not impossible to eliminate all control risk posed by humans, such as employee mistakes, collusion, or faulty decisions.

In an integrated audit of a nonissuer, each of the following identifies an inherent limitation to internal control, except: breakdowns in internal control because of employee mistakes. collusion involving two or more employees. faulty decision making by employees. an override of internal controls by a low-level employee.

II only In all audits, the auditor should obtain an understanding of each of the five components of internal control sufficient to assess the risks of material misstatement of the financial statements and to design and perform further audit procedures. The auditor must perform procedures to understand the design of controls relevant to an audit of financial statements, and whether they have been placed in operation. Whether a control has been placed in operation is different from its operating effectiveness. In obtaining knowledge about whether controls have been placed in operation, the auditor determines that the entity is using them. Operating effectiveness, on the other hand, is concerned with how the control was applied, the consistency with which it was applied, and by whom it was applied.

In obtaining an understanding of an entity's internal control, an auditor is required to obtain knowledge about the: operating effectiveness of policies and procedures. design of policies and procedures. Both I and II II only I only Neither I nor II

one year after the date the financial statements are issued or available to be issued.

In performing an audit of a nonissuer in accordance with generally accepted auditing standards, an auditor has responsibility for identifying and responding to risks that there is substantial doubt about the entity's ability to continue as a going concern for: one year after the date the financial statements are issued or available to be issued. 18 months after the balance sheet date. no more than one year after the date of the financial statements. one year after the balance sheet date.

identify the types of potential misstatements that could occur. The auditor must first obtain an understanding of the design of the entity's internal controls (the five components of internal control) and then determine if the controls have been placed in operation in order to plan the audit. The preliminary understanding is that which the question addresses. This understanding is necessary in order to: identify types of potential misstatements, consider factors that affect the risk of material misstatement, and design the nature, timing, and extent of further audit procedures. It will also be useful to the auditor later when designing tests of controls and substantive tests. The auditor then will evaluate the operating effectiveness of the internal controls in order to assess control risk, which should be documented in the audit workpapers. Assessing the operating efficiency (effectiveness) of internal control is a different step from understanding the design of the controls. Determining whether the controls have been circumvented by collusion is not an audit step; although testing of controls is necessary in order to assess the level of control risk and then determine the nature, timing, and extent of substantive tests. Documenting the assessed level of control risk comes after the evaluation of the operating effectiveness of the controls.

In planning an audit, the auditor's knowledge about the design of relevant control activities should be used to: identify the types of potential misstatements that could occur. assess the operational efficiency of internal control. determine whether controls have been circumvented by collusion. document the assessed level of control risk.

reported to the entity's governing body and the governing body fails to make a required report to the federal inspector general. Auditors are not ordinarily responsible for reporting fraud or noncompliance with laws and regulations to a party other than the entity's senior management. Specific exceptions are provided for audits of entities that receive financial assistance from a government agency (AU-C 240.A73, AU-C 250.05). In the event that the auditor reports the misappropriation of assets to the entity's governing body and the governing body fails to report to the federal inspector general, the auditor would likely be required to report the event directly to the federal inspector general.

In reporting under Government Auditing Standards, an auditor most likely would be required to communicate management's misappropriation of assets directly to a federal inspector general when the fraudulent activities are: concealed by management by circumventing specific internal controls designed to safeguard those assets. reported to the entity's governing body and the governing body fails to make a required report to the federal inspector general. accompanied by fraudulent financial reporting that results in material misstatements of asset balances. perpetrated by several levels of management in a scheme that is likely to continue in future years.

increase the emphasis on professional skepticism when gathering and evaluating audit evidence with the audit team. "Increase the emphasis on professional skepticism when gathering and evaluating audit evidence with the audit team" is the correct answer choice. Professional skepticism is an attitude that includes a questioning mind and a critical assessment of audit evidence. Greater risk of material misstatement due to fraud, errors, or other irregularities requires the auditor or audit team to exercise even more professional skepticism during the conduct of their audit. The remaining answer responses—making no changes to audit procedures, performing more procedures at an interim date, and eliminating the need for substantive tests—represent actions an auditor might take if risk of material misstatement is assessed low (or at a minimum).

In response to an increased level of assessed risk of material misstatement, an auditor of a nonissuer would generally: increase the emphasis on professional skepticism when gathering and evaluating audit evidence with the audit team. perform additional tests of controls at an interim date to eliminate the need for substantive tests at period-end. not make changes to the nature, timing, or extent of further audit procedures. perform more substantive audit procedures at an interim date instead of at period-end.

evaluating whether changes from the prior period are appropriate in the circumstances. The auditor should identify the circumstances that require accounting estimates; evaluate whether the methods for making estimates (that could be materially misstated individually or in the aggregate) are appropriate and have been applied consistently; determine if changes from the prior period are appropriate in the circumstances; and, when appropriate, test the data, methods, and significant assumptions used to develop the accounting estimates.

In terms of accounting estimates, the financial statement auditor is responsible for: evaluating whether estimates are consistently applied, regardless of the circumstances. evaluating whether changes from the prior period are appropriate in the circumstances. ensuring disclosures of all individual areas of estimation and methods of estimation are made, regardless of materiality. capturing all information to take responsibility for making necessary accounting estimates on behalf of management.

Conditions that cause the auditor to have substantial doubt about the entity's ability to continue as a going concern are inadequately disclosed. If there are inadequately disclosed conditions that cause the auditor to have substantial doubt about the entity's ability to continue as a going concern, the auditor must determine whether to issue a qualified or adverse opinion. When there is a departure from GAAP, the auditor must decide whether to issue either a qualified opinion or an adverse opinion.

In which of the following situations would an auditor ordinarily choose between expressing a qualified opinion or an adverse opinion? The auditor did not observe the entity's physical inventory and is unable to become satisfied about its balance by other auditing procedures. Conditions that cause the auditor to have substantial doubt about the entity's ability to continue as a going concern are inadequately disclosed. There has been a change in accounting principles that has a material effect on the comparability of the entity's financial statements. The auditor is unable to apply necessary procedures concerning an investor's share of an investee's earnings recognized on the equity method.

segregation of functional responsibilities in asset management. The following categories are reflected in the definition of internal control: Reliability of financial reporting Effectiveness and efficiency of operations Compliance with applicable laws and regulations Segregation of functional responsibilities is a basic concept of internal control, but not a primary objective as provided in the definition.

Internal control is a process, effected by an entity's board of directors, management, and other personnel, designed to provide reasonable assurance regarding the achievement of objectives in all of the following categories, except: reliability of financial reporting. effectiveness and efficiency of operations. segregation of functional responsibilities in asset management. compliance with applicable laws and regulations

Malone CPA can bid on and tentatively accept the engagement as long as acceptance is not finalized until inquiries of the predecessor have been completed. An auditor may make a proposal for an engagement before communicating with the predecessor auditor and can tentatively accept the engagement as long as the client is aware that acceptance cannot be finalized until the inquiries of the predecessor have been completed.

Malone CPA wants to propose on a prospective audit engagement but is unable to communicate with the predecessor auditor before the bid proposal is due. What are the implications on Malone CPA's ability to bid on this engagement? Malone CPA is prohibited from bidding on the engagement. Malone CPA can bid on and accept the engagement without making any inquiries of the predecessor auditor, as long as communications will be made prior to the start of fieldwork. Malone CPA can bid on the engagement but give no indications of even tentative acceptance until required predecessor inquiries have been completed. Malone CPA can bid on and tentatively accept the engagement as long as acceptance is not finalized until inquiries of the predecessor have been completed.

prior-year checks listed in the cutoff statement to the year-end outstanding checklist. A cutoff bank statement is a record of transactions for a specific period (less than the full-month reporting period) that is requested by the auditor from the bank. For example, the auditor may request a statement of transactions from January 1 through January 21, 20X1, when the client's year-end was December 31, 20X0. Using this statement, the auditor can compare the list of outstanding checks as of December 31, 20X0, to the checks that cleared on the cutoff statement to see if those same checks are still outstanding. Any checks dated (or written) after year-end that clear the cutoff statement would not appear on the year-end outstanding check list. Deposits in transit at the end of the year are those deposited per the books that did not yet clear the bank. The auditor would start with the year-end bank reconciliation's deposits in transit and trace to the cutoff statement, not the other way around. Any deposits recorded in the cash receipts journal (on the books) after year-end should appear on the bank statements in the next year. The auditor would not be concerned about tracing these deposits to the cutoff statement.

On receiving a client's bank cutoff statement, an auditor most likely would trace: prior-year checks listed in the cutoff statement to the year-end outstanding checklist. deposits in transit listed in the cutoff statement to the year-end bank reconciliation. checks dated after year-end listed in the cutoff statement to the year-end outstanding checklist. deposits recorded in the cash receipts journal after year-end to the cutoff statement.

Reference to the responsibility that the accountant is taking related to any supplementary information that management has elected to accompany the financial statements The engagement letter should address the following: 1. Material departures from the applicable financial reporting framework may exist, and the effects of those departures, if any, on the financial statements may not be disclosed. 2. Whether substantially all disclosures (and statement of cash flows, if applicable) required by the applicable financial reporting framework will be omitted at the election of management. 3. Reference to supplementary information or required supplementary information, if any

Patel CPA is entering into a new engagement to compile financial statements of a nonissuer. Which of the following is an element that Patel CPA should ensure is included in the letter? Reference to the responsibility that the accountant is taking related to any supplementary information that management has elected to accompany the financial statements Listing of any known material departure from the applicable financial reporting framework Patel CPA's election to substantially omit all disclosures All material departures from the applicable financial reporting framework will be identified.

Disclose such omission in the financial statements If the accountant prepares financial statements that omit substantially all disclosures required by the applicable financial reporting framework, the accountant should disclose such omission in the financial statements. The accountant should not prepare financial statements that omit substantially all disclosures required by the applicable financial reporting framework if the accountant becomes aware that the omission was undertaken with the intention of misleading users of the financial statements.

Remy CPA is engaged to prepare financial statements that substantially omit all disclosures. What are the requirements under the Statements on Standards for Accounting and Review Services (SSARS) for this type of engagement? Disclose such omission in the financial statements Disclose a known departure from the applicable financial reporting framework if the omission is being elected in order to mislead financial statement users Only a compilation engagement may be performed in this type of engagement. Issue a preparation of financial statements report, noting the omission

written report describing each significant deficiency observed, including identification of those considered material weaknesses. GAS 4.19 states, "When providing an opinion or a disclaimer on financial statements, auditors should also report on internal control over financial reporting and on compliance with provisions of laws, regulations, contracts, or grant agreements that have a material effect on the financial statements." The written report should describe each significant deficiency observed, and must include identification of those considered material weaknesses.

Reporting on internal control under Government Auditing Standards differs from reporting under generally accepted auditing standards in that Government Auditing Standards require a: statement of positive assurance that internal control procedures designed to detect material errors and fraud were tested. written report describing each significant deficiency observed, including identification of those considered material weaknesses. statement of negative assurance that internal control procedures not tested have an immaterial effect on the entity's financial statements. written report describing the entity's internal control procedures specifically designed to prevent fraud, abuse, and illegal acts.

The significant judgment involved in the interpretation of the accounting principles surrounding revenue recognition. Due to the significant judgment involved in the interpretation of the accounting principles surrounding revenue recognition and the accounting estimates often involved, the risk of material misstatement may be greater for improper revenue recognition. AU-C 315.A150

Risk of a material misstatement of recognized revenue is likely to be high due to which of the following? The significant judgment involved in the interpretation of the accounting principles surrounding revenue recognition. More transactions center around the revenue accounts than any other accounts in the chart of accounts. Both the significant judgment involved in the interpretation of the accounting principles surrounding revenue recognition, and more transactions center around the revenue accounts than any other accounts in the chart of accounts. Recognized revenue does not have a high risk of material misstatement.

Completeness The risk with contingent liabilities is that they were not recorded or understated (i.e., completeness and valuation assertions only). Only the completeness assertion answers the question, "Have all contingent liabilities been reported?" Since contingent liabilities are often related to outstanding legal issues, the most effective test is to read the board of directors' minutes, terms of significant contracts, inquire of management and legal counsel for completeness.

Russell CPA is auditing contingent liabilities. What generally is the primary risk of material misstatement related to related financial statement disclosures? Completeness Existence Rights and obligations Cutoff

40% of federal expenditures. If the auditee meets the criteria for a low-risk auditee, the auditor only needs to audit the major programs that, in aggregate, encompass at least 20% of total federal awards expended. Otherwise, the auditor must audit the major programs that, in aggregate, encompass at least 40% of total federal awards expended.

The OMB's Uniform Guidance rules contain a "percentage of coverage" rule that requires the auditor of entities receiving federal financial support who do not meet the criteria for low risk to test major programs that, in the aggregate, encompass at least: 100% of federal expenditures. 50% of federal expenditures. 40% of federal expenditures. 30% of federal expenditures

consideration of the entity's internal control has been completed. The audit program usually cannot be finalized until the consideration of the entity's internal control has been completed. An audit program should reflect the results of planning considerations and procedures. To reflect those results, such considerations must be complete. In planning the audit, the CPA should consider the anticipated reliance on internal control. The engagement letter, communicating material weaknesses, and searching for unrecorded liabilities are all necessary to finalize the audit report.

The audit program usually cannot be finalized until the: consideration of the entity's internal control has been completed. engagement letter has been signed by the auditor and the client. material weaknesses have been communicated to the audit committee of the board of directors. search for unrecorded liabilities has been performed and documented.

substantive procedures. In evaluating whether management has identified all accounting estimates that could be material to the financial statements, the auditor considers the circumstances of the industry or industries in which the entity operates, its methods of conducting business, new accounting pronouncements, and other external factors. The auditor should consider performing the following procedures: Consider assertions embodied in the financial statements to determine the need for estimates. Evaluate information obtained in performing other procedures, such as the following:Information about changes made or planned in the entity's business, including changes in operating strategy, and the industry in which the entity operates that may indicate the need to make an accounting estimateChanges in the methods of accumulating informationInformation concerning identified litigation, claims, and assessments, and other contingenciesInformation from reading available minutes of meetings of stockholders, directors, and appropriate committeesInformation contained in regulatory or examination reports, supervisory correspondence, and similar materials from applicable regulatory agencies Inquire of management about the existence of circumstances that may indicate the need to make an accounting estimate.

The auditor considers the following procedures among others in evaluating whether management has identified all accounting estimates that could be material to the financial statements except for: substantive procedures. inquiry of management. information from available minutes. changes made to the entity's business, including operating strategy, and the industry in which the entity operates.

Both I and II AU-C 330.A9 states, "Because effective internal controls generally reduce but do not eliminate the risk of material misstatement, tests of controls reduce but do not eliminate the need for substantive procedures." AU-C 330.18 states, "Irrespective of the assessed risks of material misstatement, the auditor should design and perform substantive procedures for all relevant assertions related to each material class of transactions, account balance, and disclosure." While the auditor can elect to use only a substantive approach, if a test of controls is not efficient, the auditor cannot use only a test of controls approach. Therefore, "Either I or II" is not a correct answer.

The auditor determines that effective internal controls exist for all relevant assertions related to a material class of transactions, account balance, and disclosure. As a result, the auditor can elect to perform which of the following tests? Tests of controls Substantive procedures Analytical procedures Either I or II Both I and II I only III only

The nature of the assertion tested Analytical procedures may be effective and efficient tests for assertions in which potential misstatements would not be apparent from an examination of the detailed evidence or in which detailed evidence is not readily available. The expected effectiveness and efficiency of an analytical procedure in identifying potential misstatements depends on, among other things: the nature of the assertion, the plausibility and predictability of the relationship, the availability and reliability of the data used to develop the expectation, and the precision of the expectation.

The auditor should consider certain factors in assessing the efficiency and effectiveness of analytical procedures as compared to tests of details. In determining whether and to what extent analytical procedures should be used, which of the following should the auditor consider? Interrelationships of financial information Nonfinancial information that may affect financial information The nature of the assertion tested Explanations provided by the client

A definition of internal control PCAOB AS 2201.85 states that the auditor's report on the audit of internal control over financial reporting must include a definition of internal control which is the same description of the entity's internal control as management uses in its report. The other items listed are reporting elements for an agreed-upon procedures report on controls.

The auditor's report on an integrated audit of internal control for an issuer should include which of the following? Identification of the specified parties A statement that the sufficiency of the procedures is solely the responsibility of the specified parties and a disclaimer of responsibility for the sufficiency of those procedures A list of the procedures performed and related findings A definition of internal control

The auditor should perform limited procedures. Supplementary information is defined by AU-C 725 as "information presented outside the basic financial statements, excluding required supplementary information that is not considered necessary for the financial statements to be fairly presented in accordance with the applicable financial reporting framework. Such information may be presented in a document containing the audited financial statements or separate from the financial statements." The auditor should perform limited procedures, including the following: Inquire of management about the purpose of the supplementary information, the criteria used by management to prepare the information, and any significant assumptions or interpretations underlying the measurement or presentation of the information Compare and reconcile the information to the underlying accounting and other records used in preparing the financial statements Inquire of management about any significant assumptions or interpretations Perform procedures to test the completeness and accuracy of the information Obtain written representations from management regarding the information The auditor should report on supplementary information either in an other-matter paragraph in the auditor's report on the financial statements or in a separate report on the supplementary information.

The client's financial reporting includes supplementary financial information outside the basic financial statements but required by the Financial Accounting Standards Board (FASB). Which of the following statements is correct regarding the auditor's responsibility for this supplementary financial information? The auditor should perform limited procedures. The auditor is not required to report omissions. The auditor should read the supplementary financial information. The auditor should apply tests of details of transactions.

independent for purposes of examining financial information required to be filed annually with the DOL. The Employee Retirement Income Security Act (ERISA) regulates employers who offer pension or welfare benefit plans. According to ERISA, auditors of employee benefit plans must be independent, in that they should not have any financial interests in the plan or the plan sponsor that would affect their ability to render an objective, unbiased opinion about the financial condition of the plan.

The controller of a small utility company has interviewed audit firms proposing to perform the annual audit of their employee benefit plan. According to the guidelines of the Department of Labor (DOL), the selected auditor must be: the firm that proposes the lowest fee for the work required. independent for purposes of examining financial information required to be filed annually with the DOL. included on the list of firms approved by the DOL. independent of the utility company and not relying on its services.

That the engagement quality control reviewer may make recommendations to the engagement partner but must defer to his or her judgment when differences of opinion exist An engagement quality control review may include consideration of the following: The engagement team's evaluation of the firm's independence in relation to the specific engagement Whether appropriate consultation has taken place on matters involving differences of opinion or other difficult or contentious matters and the conclusions arising from those consultations Whether documentation selected for review reflects the work performed in relation to the significant judgments and support the conclusions reached QC 10.A42 The engagement quality control reviewer is not required to defer to the engagement partner when differences of opinion exist. Instead, the firm's procedures for dealing with such differences apply.

The firm should establish criteria against which all engagements are to be evaluated to determine whether an engagement quality control review should be performed. Which of the following is not a consideration included in an engagement quality control review? The engagement team's evaluation of the firm's independence in relation to the specific engagement Whether appropriate consultation took place on matters involving differences of opinion or other difficult or contentious matters and the conclusions arising from those consultations That the engagement quality control reviewer may make recommendations to the engagement partner but must defer to his or her judgment when differences of opinion exist Whether working papers selected for review reflect the work performed in relation to the significant judgments and support the conclusions reached

does not support the auditor's planned assessed level of control risk when the true operating effectiveness of internal control justifies such an assessment. The risk of assessing control risk too high is the risk that the assessed level of control risk based on the sample is greater than the true operating effectiveness of the control. In other words, the sample tested by the auditor has a higher rate of deviation than the full population does. Based on the testing, the auditor assesses control risk higher than he or she would if the auditor had tested the whole population.

The likelihood of assessing control risk too high is the risk that the sample selected to test controls: does not support the auditor's planned assessed level of control risk when the true operating effectiveness of internal control justifies such an assessment. contains misstatements that could be material to the financial statements when aggregated with misstatements in other account balances or transactions classes. contains proportionately fewer monetary errors or deviations from prescribed internal control structure policies or procedures than exist in the balance or class as a whole. does not support the tolerable error for some or all of management's assertions.

identifies the applicable financial reporting framework and its origin. The answer choice "identifies the applicable financial reporting framework and its origin" is correct. The auditor should state that the financial statements present fairly, in all material respects, the financial position of the entity as of the balance sheet date and the results of its operations and its cash flows for the period then ended in accordance with the applicable financial reporting framework and its origin, which is typically U.S. GAAP. The answer choice "includes the word 'independent' to clearly indicate that the report is from an independent auditor" is incorrect as this language is found in the title of the auditor's report. The answer choice "describes the auditor's responsibility for expressing an opinion on the financial statements" is incorrect because this language is found in the auditor's responsibility section of the auditor's report. The answer choice "indicates that management is responsible for the fair presentation of the financial statements" is incorrect as this language is incorporated into the management's responsibility for the financial statements paragraph of the auditor's report.

The opinion paragraph in an auditor's report for a nonissuer should include a statement that: includes the word "independent" to clearly indicate that the report is from an independent auditor. describes the auditor's responsibility for expressing an opinion on the financial statements. identifies the applicable financial reporting framework and its origin. indicates that management is responsible for the fair presentation of the financial statements.

authorization of transactions from the custody of related assets. Control activities are the policies and procedures that help ensure management directives are carried out. Specific control activities include the following: -Authorization -Performance reviews -Information processing -Physical controls -Segregation of duties When someone has the ability to perform more than one of the control activities, there is a violation of the segregation of duties. Segregation of duties (SOD) is a basic building block of risk management and internal controls for a business. The principle of SOD is based on shared responsibilities of a key process that disperses the critical functions of that process to more than one person or department. Without this separation in key processes, fraud and error risks are far less manageable. By having control over the authorization of transactions from the custody of transaction, key processes are separated as defined above.

The purpose of segregating the duties of hiring personnel and distributing payroll checks is to separate the: human resources function from the controllership function. administrative controls from the internal accounting controls. authorization of transactions from the custody of related assets. operational responsibility from the recordkeeping responsibility.

a paragraph describing management's assertion concerning the effectiveness of internal control. The written communication regarding significant deficiencies and material weaknesses identified during the audit of financial statements should include: the definition of the term "material weakness" and, when relevant, the definition of the term "significant deficiency." a description of the significant deficiencies and material weaknesses and an explanation of their potential effects. sufficient information to enable those charged with governance and management to understand the context of the communication. In particular, the auditor should include in the communication the following elements that explain that the purpose of the audit was for the auditor to express an opinion on the financial statements: The audit included consideration of internal control over financial reporting in order to design audit procedures that are appropriate in the circumstances but not for the purpose of expressing an opinion on the effectiveness of internal control. The auditor is not expressing an opinion on the effectiveness of internal control. The auditor's consideration of internal control was not designed to identify all deficiencies in internal control that might be material weaknesses or significant deficiencies, and therefore, material weaknesses or significant deficiencies may exist that were not identified. A paragraph describing management's assertion concerning the effectiveness of internal control is not a required communication.

The written communication regarding significant deficiencies and material weaknesses identified during the audit should include all of the following except: a statement that the purpose of the auditor's consideration of internal control was to express an opinion on the financial statements, but not to express an opinion on the effectiveness of the entity's internal control. the definition of the term "material weakness" and, where relevant, the definition of the term "significant deficiency." a paragraph describing management's assertion concerning the effectiveness of internal control. the matters that are considered to be significant deficiencies and those that are considered to be material weaknesses.

There are two types of reports that a service organization, in this case PDC, may request from its auditor ("service auditor"): a report on management's description of a service organization's system and the suitability of the design of controls (also referred to as a type 1 report) and a report on management's description of a service organization's system and the suitability of the design and operating effectiveness of controls (also referred to as a type 2 report). note: Think D1 = Design of controls = type 1 Think tO = Operating effectiveness of controls = Type 2

There are two types of reports that a service organization, in this case PDC, may request from its auditor ("service auditor"): a report on management's description of a service organization's system and the suitability of the design of controls (also referred to as a _______ report) and a report on management's description of a service organization's system and the suitability of the design and operating effectiveness of controls (also referred to as a ______ report).

Sufficient to assess the risk of material misstatement and design the detailed audit plan The auditor should obtain an understanding of control activities relevant to the audit. An audit does not require an understanding of all control activities. The auditor's emphasis is on identifying and obtaining an understanding of control activities that address the areas where the auditor considers that material misstatements are more likely to occur.

To what extent is an auditor required to gain an understanding of the control activities component of internal control when satisfying obligations to understand the entity and its environment in a financial statement audit of a nonissuer? Sufficient to assess the risk of material misstatement and design the detailed audit plan Sufficient to understand the design and implementation of all control activities relevant to the entity's accounting and financial reporting systems Sufficient to identify material weaknesses in operating effectiveness Sufficient to prevent and detect fraud and noncompliance with laws and regulations

The company issues financial statements that purport to present financial position and results of operations, but refuses to include the related statement of cash flows. A disclaimer of opinion would be appropriate if there are scope issues significant enough to prevent the auditor from forming an opinion on the fairness of presentation of the financial statements in conformity with an applicable financial reporting framework. The only answer choice that does not represent a significant scope issue is "the company issues financial statements that purport to present financial position and results of operations, but refuses to include the related statement of cash flows." This situation is a type of departure from an applicable financial reporting framework because the financial statements do not include all of the disclosures (statement of cash flows) required by the applicable financial reporting framework. The auditor should issue a qualified opinion.

Under which of the following circumstances would the expression of a disclaimer of opinion be inappropriate? The chief financial officer and the chief executive officer are unwilling to sign the management representation letter. The auditor is unable to determine the extent of or the amounts associated with a pervasive employee fraud scheme. Management refuses to produce documentation verifying the ownership of its equipment and production facilities. The company issues financial statements that purport to present financial position and results of operations, but refuses to include the related statement of cash flows.

West should disclose the basis of accounting used in West's compilation report. Financial statements prepared in accordance with a special-purpose framework are not considered appropriate in form unless the financial statements include: A description of the framework, including a summary of significant accounting policies and a description of the primary differences from generally accepted accounting principles (GAAP). The effects of the differences need not be quantified. Informative disclosures similar to those required by GAAP if the financial statements contain items that are the same as, or similar to, those in financial statements prepared in accordance with GAAP. Thus, the differences need not be quantified. An entity may request the accountant to compile financial statements that omit substantially all the disclosures required by an applicable financial reporting framework, including disclosures that might appear in the body of the financial statements. The accountant may compile such financial statements, provided that the omission of substantially all disclosures is not, to his or her knowledge, undertaken with the intention of misleading those who might reasonably be expected to use such financial statements. When reporting on financial statements that omit substantially all disclosures, the accountant should include, after the paragraph describing the accountant's responsibility, a paragraph in the compilation report that includes the following elements: A statement that management has elected to omit substantially all the disclosures (and the statement of cash flows, if applicable) required by the applicable financial reporting framework (or ordinarily included in the financial statements if the financial statements are prepared in accordance with a special-purpose framework) A statement that if the omitted disclosures (and statement of cash flows, if applicable) were included in the financial statements, they might influence the user's conclusions about the company's financial position, results of operations, and cash flows (or equivalent for presentations other than accounting principles generally accepted in the United States of America) A statement that, accordingly, the financial statements are not designed for those who are not informed about such matters Since the financial statements and accompanying footnotes are the responsibility of Lake's management and Lake's management has elected to omit substantially all disclosures, the basis of accounting should be disclosed in West's compilation report.

West, CPA, is engaged to compile the financial statements of Lake Co., a nonissuer. Lake's financial statements are prepared in conformity with the cash basis of accounting. If Lake's financial statements do not disclose the basis of accounting used, which of the following statements best describes West's reporting responsibility concerning this matter? West should disclose the basis of accounting used in the notes to Lake's financial statements. West should disclose the basis of accounting used in West's compilation report. West should quantify the effects of the differences between GAAP and the cash basis and disclose them in West's compilation report. West should quantify the effects of the differences between GAAP and the cash basis and disclose them in the notes to Lake's financial statements.

state that the CPA performed procedures to evaluate management's assumptions. Financial statements can be projected into future years if certain assumptions are made. These assumptions include percent increases in sales, staffing, etc., and are made by management. The CPA needs to perform procedures to evaluate these assumptions. The CPA does not, however, need to explain the differences between historical and projected financial statements. Also, because projected financial statements are not historical, the CPA does not need to refer to the auditor's report or include an opinion on the entity's ability to continue as a going concern.

When a CPA examines a client's projected financial statements, the CPA's report should: explain the principal differences between historical and projected financial statements. state that the CPA performed procedures to evaluate management's assumptions. refer to the CPA's auditor's report on the historical financial statements. include the CPA's opinion on the client's ability to continue as a going concern.

internal control policies and procedures may be ineffective due to mistakes in judgment and personal carelessness. No matter how well internal controls may be designed and operated, they can only provide an entity with reasonable, but not absolute, assurance about achieving the entity's objectives. Certain limitations are inherent to internal control, including the following: Human judgment in decision making can be faulty. Breakdowns in internal control can occur because of human failures such as simple errors or mistakes. Errors may occur in the use of information produced by IT when that information is not effectively used because the individual responsible for reviewing the information does not understand its purpose or fails to take appropriate action. Controls can be circumvented by collusion or inappropriate management override. Management may make judgments on the nature and extent of the controls it chooses to implement and the nature and extent of the risk it chooses to assume.

When considering internal control, an auditor should be aware of the concept of reasonable assurance, which recognizes that: internal control policies and procedures may be ineffective due to mistakes in judgment and personal carelessness. adequate safeguards over access to assets and records should permit an entity to maintain proper accountability. establishing and maintaining internal control is an important responsibility of management. the cost of an entity's internal control should not exceed the benefits expected to be derived.

internal control policies and procedures may be ineffective due to mistakes in judgment and personal carelessness. No matter how well internal controls may be designed and operated, they can only provide an entity with reasonable, but not absolute, assurance about achieving the entity's objectives. Certain limitations are inherent to internal control, including the following: Human judgment in decision making can be faulty. Breakdowns in internal control can occur because of human failures such as simple errors or mistakes. Errors may occur in the use of information produced by IT when that information is not effectively used because the individual responsible for reviewing the information does not understand its purpose or fails to take appropriate action. Controls can be circumvented by collusion or inappropriate management override. Management may make judgments on the nature and extent of the controls it chooses to implement and the nature and extent of the risk it chooses to assume.

When considering internal control, an auditor should be aware of the concept of reasonable assurance, which recognizes that: internal control policies and procedures may be ineffective due to mistakes in judgment and personal carelessness. adequate safeguards over access to assets and records should permit an entity to maintain proper accountability. establishing and maintaining internal control is an important responsibility of management. the cost of an entity's internal control should not exceed the benefits expected to be derived.

Reevaluate the risk of fraud, and design alternate tests for the related transactions The reliability of audit evidence is influenced by its source and by its nature and is dependent on the individual circumstances under which it is obtained. Internally generated audit evidence, as with the photocopies, is more reliable when the related controls imposed by the entity are effective. Reevaluating the risk of fraud and designing alternate tests will help the auditor determine the reliability of the audit evidence. Increasing the number of items tested does not increase the reliability because the evidence for one vendor is still photocopies and counting the document as a misstatement may not be a valid classification.

When performing a substantive test of a random sample of cash disbursements, an auditor is supplied with a photocopy of vendor invoices supporting the disbursements for one particular vendor rather than the original invoices. The auditor is told that the vendor's original invoices have been misplaced. What should the auditor do in response to this situation? Increase randomly the number of items in the substantive test to increase the reliance that may be placed on the overall test Reevaluate the risk of fraud, and design alternate tests for the related transactions Increase testing by agreeing more of the payments to this particular vendor to the photocopies of its invoices Count the missing original documents as misstatements, and project the total amount of the error based on the size of the population and the dollar amount of the errors

Program change control Input controls, processing controls, and output controls are all examples of application controls. Examples of general controls are program change controls, controls that restrict access to programs or data, controls over the implementation of new releases of packaged software applications, and controls over system software that restrict access to or monitor the use of system utilities that could change financial data or records without leaving an audit trail.

Which of the following are examples of general controls in an IT environment? Input controls Processing controls Program change control Output controls

An attestation engagement to examine and report on management's written assertions about the effectiveness of its internal control The attestation standards define an attest engagement as "one in which a practitioner is engaged to issue or does issue an examination, a review, or an agreed-upon procedures report on subject matter, or an assertion about the subject matter, that is the responsibility of another party." A consulting engagement to provide constructive advice to the entity on its internal control would not provide any assurance. An audit of internal controls would only be appropriate if integrated with a financial statement audit.

Which of the following best describes a CPA's examination engagement to report on an entity's internal control over financial reporting that is not integrated in a financial statement audit? An attestation engagement to examine and report on management's written assertions about the effectiveness of its internal control An audit engagement to render an opinion on the entity's internal control A prospective engagement to project, for a period of time not to exceed one year, and report on the expected benefits of the entity's internal control A consulting engagement to provide constructive advice to the entity on its internal control

Management adheres to internal control policies. Internal control consists of five interrelated components: (1) control environment, (2) risk assessment, (3) information and communication, (4) control activities, and (5) monitoring. The control environment sets the tone of an organization, influencing the control consciousness of its people. It is the foundation for all other components of internal control, providing discipline and structure. The internal control that contributes the most to a strong control environment is the attitude, awareness, and actions of those charged with governance, particularly management. They set the "tone at the top," which has a trickle-down effect to all levels of employees in terms of establishing and maintaining a strong internal control environment.

Which of the following components of internal control contributes most to a strong control environment? Controls are assessed through ongoing activities and evaluations. Management adheres to internal control policies. Duties are clearly defined and separated. Policy manuals provide a clear understanding of internal controls.

Select more effective substantive tests If an auditor concludes that there is a high risk of material misstatement, the auditor may expand substantive testing or select more effective substantive tests. Using smaller sample sizes, performing substantive tests at an interim date, and increasing tests of controls are not examples of expanding substantive testing or selecting more effective substantive tests.

Which of the following courses of action is the most appropriate if an auditor concludes that there is a high risk of material misstatement? Use smaller, rather than larger, sample sizes Perform substantive tests as of an interim date Select more effective substantive tests Increase of tests of controls

Personnel The personnel department is charged with planning, controlling, and coordinating employees. This will determine the proposed salary changes, and ensure they are aligned to the company salary guidelines and public benchmarks.

Which of the following departments most likely would approve changes in pay rates and deductions from employee salaries? Personnel Treasurer Controller Payroll

An integrated audit of internal control over financial reporting and audit of financial statements An engagement quality control review is required for financial statement and integrated internal control audits of issuers. The engagement quality control reviewer should evaluate significant judgments made by the engagement team and the related conclusions reached.

Which of the following engagements would always have an engagement quality control review? Audits of employee benefit plans All financial statement audits An integrated audit of internal control over financial reporting and audit of financial statements Single audits

An integrated audit of internal control over financial reporting and audit of financial statements An engagement quality control review is required for financial statement and integrated internal control audits of issuers. The engagement quality control reviewer should evaluate significant judgments made by the engagement team and the related conclusions reached.

Which of the following engagements would always have an engagement quality control review? Audits of employee benefit plans All financial statement audits An integrated audit of internal control over financial reporting and audit of financial statements Single audits

The adequacy of the accounting records The determination of the auditability of an entity is one of the first matters the auditor considers in the initial planning for an audit engagement and before accepting it. One of the requisites of auditability is the availability of adequate, i.e., sufficient, reliable accounting records. Without adequate accounting records, auditors may not be able to obtain sufficient appropriate audit evidence to support an opinion on the entity's financial statements. In planning the audit, the auditor considers the methods used by the entity to accumulate, process, and review accounting information because such methods influence the design of internal control, another critical audit concern. The other answer choices—the complexity of the accounting system, the existence of related party transactions, and the operating effectiveness of controls—relate more to the auditor's consideration of the nature, timing, and extent of audit procedures that will be required (e.g., whether any extension or modification of audit testing will be necessary).

Which of the following factors most likely would influence an auditor's determination of the auditability of an entity's financial statements? The complexity of the accounting system The existence of related party transactions The adequacy of the accounting records The operating effectiveness of controls

The tolerable misstatement Attribute sampling is primarily utilized when conducting a sample for the purposes of evaluating internal controls. The variable of interest is an attribute such as the presence of a sign-off on a document as evidence of its review or evidence of approval of a journal entry. Because attribute sampling is not performed to test dollar balances in an account, tolerable misstatement is not a factor that is considered when calculating the sample size for attribute sampling.

Which of the following factors would the auditor not explicitly consider when determining sample size in an attribute sample for a test of controls? An acceptable level of the risk of overreliance The tolerable misstatement The expected population deviation rate The tolerable deviation rate

Department of Labor independence rules The AICPA, DOL (Department of Labor), and SEC (Securities and Exchange Commission) all have rules regarding auditor independence. The DOL rules apply to all employee benefit plan auditors, the AICPA rules also apply to those employee benefit plan auditors who are members of the AICPA, and the SEC's rules apply to auditors of employee benefit plans that file on Form 11-K with the SEC.

Which of the following independence rules apply to all employee benefit plan auditors, regardless of the nature of the employee benefit plan being audited? Department of Labor independence rules Government Auditing Standards independence rules Securities and Exchange Commission independence rules Public Company Accounting Oversight Board independence rules

Approving vendors' invoices for payment The following control activities are policies and procedures that help ensure that management directives are carried out and necessary actions are taken to address risk that threaten achievement of the entity's objectives: Authorization Segregation of duties Safeguarding/physical controls Asset accountability/information processing Performance reviews The treasurer's department should not approve vendor's invoices for payment as it would put the treasurer's department in control of both authorization and information processing. This would not be a proper segregation of duties.

Which of the following internal control procedures is not usually performed in the treasurer's department? Verifying the accuracy of checks and vouchers Controlling the mailing of checks to vendors Approving vendors' invoices for payment Canceling payment vouchers when paid

Establishing budgets and forecasts to identify variances from expectations The problem asks which management control method could most likely improve management's ability to supervise company activities effectively. This goal requires a control technique that communicates and monitors management's expectations regarding all of the company's activities. Monitoring compliance with internal controls imposed by regulatory bodies could improve the effectiveness in that particular area but it would not enhance management's ability to supervise the company's activities. Limiting direct access to assets would improve control over assets but this also would not improve management's ability to supervise the company's activities. Supporting employees with necessary resources would certainly aid in the accomplishment of company activities but would do nothing to help management's ability to supervise those activities. Of all the answers provided, using budgets and forecasts and monitoring variances from them is the only management control method that could most likely improve management's ability to supervise company activities effectively.

Which of the following is a management control method that most likely could improve management's ability to supervise company activities effectively? Monitoring compliance with internal control requirements imposed by regulatory bodies Limiting direct access to assets by physical segregation and protective devices Establishing budgets and forecasts to identify variances from expectations Supporting employees with the resources necessary to discharge their responsibilities

The auditor is required to assess the risk of fraud. The auditor is required to assess fraud risk in all audit engagements because the auditor provides assurance in the unmodified audit report that the financial statements are free from material misstatement due to either error or fraud. The other answer choices are incorrect because an audit of internal control is not required for nonissuers or some issuers.

Which of the following is a requirement for an audit of both an issuer's and a nonissuer's financial statements? The auditor is required to assess the risk of fraud. The auditor is required to refer to a recognized control framework in performing the audit of internal control over financial reporting. The auditor is required to express an opinion on the effectiveness of the company's internal control over financial reporting. The auditor is required to assess the effectiveness of management's assessment of the company's internal control over financial reporting.

The loss of data and information could have broader business implications in an automated environment. Authorization procedures in many computer systems are a part of the computer program. Thus, there is increased potential for unauthorized individuals to gain access to sensitive accounting information. Concerning the maintenance of adequate documentary evidence, auditors must be aware that the traditional audit trail may not be available due to the fact that the IT system does not provide a hard copy of source documents. Adequate controls must be established within the IT department to compensate for the lack of segregation of duties that would normally be available in a manual system. It is also important to have adequate backup for computer files, as the loss of data/information could have broader business implications in an automated environment versus a manual one.

Which of the following is a true statement related to using information technology in internal control over financial reporting over manual controls? The potential for unauthorized individuals to gain access to sensitive accounting information is less in an automated environment. The ability to provide an audit documentation trail is improved in an automated environment. The loss of data and information could have broader business implications in an automated environment. The need for segregation of duties is eliminated in an automated environment.

Engagement performance The CPA firm's system of quality control should include policies and procedures addressing the following elements: Leadership responsibilities for quality within the firm ("tone at the top") Relevant ethical requirements Acceptance and continuance of client relationships and specific engagements Human resources Engagement performance Monitoring

Which of the following is an element of a CPA firm's quality control policies and procedures applicable to the firm's accounting and auditing practice? Information processing Engagement performance Technology selection Professional skepticism

Maintain a times interest earned ratio of at least 6. The only covenant that is phrased correctly is time interest earned of at least 6. Times interest earned measures the ability to cover interest charges, and a minimum number of months required is common as a debt covenant. Minimum current ratios are required, as opposed to maximum ratios. Maximum debt-to-equity ratios are required, as opposed to minimum ratios. Lenders do not generally stipulate maximum profit margin requirements.

Which of the following is most likely to be included as a debt covenant for a lender that is concerned about a borrower's ability to pay back a loan when due? Maintain a maximum current ratio of 1:1. Maintain a minimum debt-to-equity ratio of 2:1. Maintain a times interest earned ratio of at least 6. Maintain a maximum profit margin on sales of 8%.

Significant conclusions reached on the engagement are discussed with the firm's peer reviewer. A review may include consideration of whether, for example: the work has been performed in accordance with professional standards and applicable regulatory and legal requirements, significant findings and issues have been raised for further consideration, appropriate consultations have taken place and the resulting conclusions have been documented and implemented, the nature, timing, and extent of work performed is appropriate and without need for revision, the work performed supports the conclusions reached and is appropriately documented, the evidence obtained is sufficient and appropriate to support the report, and the objectives of the engagement procedures have been achieved.

Which of the following is not a quality control policy or procedure related to the review of work performed by other engagement team members? The work performed supports the conclusions reached and is appropriately documented. Significant conclusions reached on the engagement are discussed with the firm's peer reviewer. The evidence obtained is sufficient and appropriate to support the report. The objectives of the engagement procedures have been achieved.

Consideration of the historical experience of the entity in making past estimates as well as the auditor's experience in the industry In evaluating the reasonableness of an estimate, the auditor normally concentrates on key factors and assumptions that are: significant to the accounting estimate, sensitive to variations, deviations from historical patterns, and subject and susceptible to misstatement and bias. The auditor should consider the historical experience of the entity in making past estimates, as well as the auditor's experience in the industry.

Which of the following key factors and assumptions would an auditor normally concentrate on when evaluating the reasonableness of an estimate? Lack of relative significance of the accounting estimate That the estimate has little susceptibility to misstatement and biasness Consideration of the historical experience of the entity in making past estimates as well as the auditor's experience in the industry That the entity has established an audit committee to oversee the audit function

Selecting a sample of vendor invoices for comparison to receiving reports Audit planning involves developing an overall strategy for the expected conduct, organization, and staffing of the audit. All of the answer choices involve steps that would be taken during audit planning except "selecting a sample of vendor invoices for comparison to receiving reports." This activity would occur during substantive testing.

Which of the following procedures would an auditor least likely perform in planning a financial statement audit? Coordinating the assistance of entity personnel in data preparation Discussing matters that may affect the audit with firm personnel responsible for nonaudit services to the entity Selecting a sample of vendor invoices for comparison to receiving reports Reading the current year's interim financial statements

Performing cutoff procedures for shipping and receiving The completeness assertion states that every transaction that should have been recorded actually was recorded, and all related disclosures that should have been included in the financial statements have been included. Testing for the completeness assertion is testing to see that the list is not complete, and performing cutoff procedures accomplishes this task. Scanning records, examining paid invoices, and tracing items from the tag listing back to the physical inventory do not determine if the item has or has not been recorded.

Which of the following procedures would be most appropriate for testing the completeness assertion as it applies to inventory? Scanning perpetual inventory, production, and purchasing records Examining paid vendor invoices Tracing inventory items from the tag listing back to the physical inventory quantities Performing cutoff procedures for shipping and receiving

Ratio of accounts payable to total current liabilities of 4:1, compared to 6:1 for the prior period Analytical procedures consist of evaluations of financial information made by a study of plausible relationships among both financial and nonfinancial data. A basic premise underlying the application of analytical procedures is that plausible relationships among data may reasonably be expected to exist and continue in the absence of known conditions to the contrary. One type of analytical procedure is ratio analysis. Assuming the numerator is held constant, a change in the current ratio from 5:1 to 2:1 and a decrease in the accounts payable turnover of from 10 to 5 would both indicate an increase in current liabilities; this is not an indication of unrecorded liabilities. An increase in the accounts payable balance over the prior period is also not an indicator of unrecorded liabilities. However, if the ratio of accounts payable to total current liabilities changed from 6:1 to 4:1, that could be an indication that the accounts payable balance for the current year was less than the prior year, and would be an indication of possible unrecorded liabilities.

Which of the following results of analytical procedures would most likely indicate possible unrecorded liabilities? Current ratio of 2:1 as compared to 5:1 for the prior period Ratio of accounts payable to total current liabilities of 4:1, compared to 6:1 for the prior period Accounts payable turnover of 5, compared to 10 for the prior period Accounts payable balance increase greater than 10% over the prior period

A client's accounting data cannot by itself be considered sufficient appropriate audit evidence to support the financial statements. AU-C 500.A7 states that accounting data, such as books of original entry, general and subsidiary ledgers, and related accounting manuals, constitute evidence in support of the financial statements. The section goes on to note that, by itself, accounting data cannot be considered sufficient support for financial statements.

Which of the following statements concerning audit evidence is correct? Appropriate evidence supporting management's assertions should be convincing rather than merely persuasive. Effective internal control contributes little to the reliability of the evidence created within the entity. The cost of obtaining evidence is an important consideration to an auditor in deciding what evidence should be obtained. A client's accounting data cannot by itself be considered sufficient appropriate audit evidence to support the financial statements.

"We believe that the action can be settled for less than the damages claimed." basically because we need to know how much less so we can include that information in the financials

Which of the following statements extracted from a client's lawyer's letter concerning litigation, claims, and assessments most likely would cause the auditor to request clarification? "We believe that the possible liability to the company is nominal in amount." "We believe that the action can be settled for less than the damages claimed." "We believe that the plaintiff's case against the company is without merit." "We believe that the company will be able to defend this action successfully."

The accountant does not contemplate obtaining an understanding of internal control. In discussion of a compilation agreement on engagement terms, AR-C 90.14 makes no mention of the accountant obtaining an understanding of internal control for either a review or a compilation. Independence is required when providing auditing or other attestation services and when performing a review. Although the accountant expresses no opinion or assurance in a compilation, he does express limited assurance in a review report. The accountant is required to obtain a written management representation letter for a review engagement (since the report will provide limited assurance).

Which of the following statements is correct concerning both an engagement to compile and an engagement to review a nonpublic entity's financial statements? The accountant does not contemplate obtaining an understanding of internal control. The accountant must be independent in fact and appearance. The accountant expresses no assurance on the financial statements. The accountant should obtain a written management representation letter.

The auditor controls the risk of incorrect acceptance by specifying that risk level for the sampling plan. Probability-proportional-to-size (PPS) sampling selects a sample based on dollars, not individual items. If the sampling interval for a particular asset is $12,000, every invoice that has a value of over $12,000 will be selected. Large-dollar-value items have a higher chance of being selected in the testing, and overstatements are more likely to be detected than understatements. Since PPS sampling is used in the test of details, the auditor would consider two types of risk: incorrect acceptance and incorrect rejection. The auditor controls those risks by determining the risk level for the sample.

Which of the following statements is correct concerning probability-proportional-to-size (PPS) sampling, also known as dollar-unit sampling? The sampling distribution should approximate the normal distribution. Overstated units have a lower probability of sample selection than units that are understated. The auditor controls the risk of incorrect acceptance by specifying that risk level for the sampling plan. The sampling interval is calculated by dividing the number of physical units in the population by the sample size.

Deviations from specific internal control procedures at a given rate ordinarily result in misstatements at a lower rate. Two types of sampling risk that affect performing tests of controls are: The risk of assessing control risk too low The risk of assessing control risk too high If the auditor assesses control risk too high, which would occur if there are deviations from an internal control procedure in the sample, the auditor would then lower detection risk by altering the nature, timing, and extent of substantive audit procedures, reducing the risk that the auditor would not detect a misstatement in the financial statements.

Which of the following statements is correct concerning statistical sampling in tests of controls? As the population size increases, the sample size should increase proportionately. Deviations from specific internal control procedures at a given rate ordinarily result in misstatements at a lower rate. There is an inverse relationship between the expected population deviation rate and the sample size. In determining tolerable rate, an auditor considers detection risk and the sample size.

Encryption performed by physically secure hardware devices is more secure than encryption performed by software. IT poses specific risks to an entity's internal control, including the following: Reliance on systems or programs that are inaccurately processing data, processing inaccurate data, or both Unauthorized access to data, including the recording of unauthorized or nonexistent transaction or inaccurate recording of transactions Possibility of IT personnel gaining access privileges beyond those necessary to perform assigned duties Unauthorized changes to data in master file Unauthorized changes to systems or programs Failure to make necessary changes to systems Inappropriate manual intervention Potential loss of data or inability to access data Encryption performed by physically secure hardware devices is more secure than encryption performed by software, and helps prevent unauthorized access to data and the possibility of IT personnel gaining access privileges.

Which of the following statements is correct concerning the security of messages in an electronic data interchange (EDI) system? When the confidentiality of data is the primary risk, message authentication is the preferred control rather than encryption. Encryption performed by physically secure hardware devices is more secure than encryption performed by software. Message authentication in EDI systems performs the same function as segregation of duties in other information systems. Security at the transaction phase in EDI systems is not necessary because problems at that level will usually be identified by the service provider.

A CPA seeking to enter into a contract to perform an audit should provide the CPA's most recent external quality control review report to the party contracting for the audit. A quality control requirement under Government Auditing Standards requires a CPA seeking to enter into an audit contract to provide the CPA's most recent external quality control review report to the party contracting for the audit.

Which of the following statements represents a quality control requirement under Government Auditing Standards? A CPA who conducts government audits is required to undergo an annual external quality control review when an appropriate internal quality control system is not in place. A CPA seeking to enter into a contract to perform an audit should provide the CPA's most recent external quality control review report to the party contracting for the audit. An external quality control review of a CPA's practice should include a review of the workpapers of each government audit performed since the prior external quality control review. A CPA who conducts government audits may not make the CPA's external quality control review report available to the public.

After performing our preliminary analytical procedures we will discuss with you the other procedures we consider necessary to complete the engagement. Of the statements listed, the least likely to appear in an auditor's engagement letter is a statement assuring the client that the auditor, after performing preliminary analytical procedures, will discuss other procedures considered necessary to complete the engagement. Among others, what the engagement letter should contain are statements regarding: the basis of the auditor's fee, the objective of the engagement and additional work to be performed such as management advisory services, and the fact that management is responsible for the entity's financial statements.

Which of the following statements would least likely appear in an auditor's engagement letter? Fees for our services are based on our regular per diem rates, plus travel and other out-of-pocket expenses. During the course of our audit, we may observe opportunities for economy in, or improved controls over, your operations. Management is responsible for the entity's financial statements. After performing our preliminary analytical procedures we will discuss with you the other procedures we consider necessary to complete the engagement.

Preliminary judgments about materiality levels When planning a particular sample for a substantive test of details, the auditor should consider the preliminary estimates of materiality levels. If, in the specific circumstances of the entity, one or more particular classes of transactions, account balances, or disclosures exist for which there is a substantial likelihood that misstatements of lesser amounts than materiality for the financial statements as a whole would influence the judgment made by a reasonable user based on the financial statements, the auditor also should determine the materiality level or levels to be applied to those particular classes of transactions, account balances, or disclosures.

Which of the following would be a consideration in planning a sample for a test of subsequent cash receipts? Preliminary judgments about materiality levels The amount of bad debt write-offs in the prior year The size of the intercompany receivable balance The auditor's allowable risk of assessing control risk being too low

The auditor's allowable risk of assessing control risk is too low. When determining a sample size for a test of controls, the auditor should consider the tolerable rate of deviation from the controls (expressed in %), the likely rate of deviations (expressed in %), and the allowable risk of assessing control risk too low (the reliability level).

Which of the following would be a consideration in planning an auditor's sample for a test of controls? Preliminary judgments about materiality levels The auditor's allowable risk of assessing control risk is too high. The level of detection for the account The auditor's allowable risk of assessing control risk is too low.

analyses of capital stock and other owners' equity accounts. The permanent workpaper file of the auditor should contain items that remain relatively unchanged from year to year. Analysis of capital stock and other owner's equity accounts represent accounts which have very few transactions each year and would be appropriate for the permanent file.

Although the quantity and content of audit documentation varies with each particular engagement, an auditor's permanent files most likely include: schedules that support the current year's adjusting entries. prior years' accounts receivable confirmations that were classified as exceptions. documentation indicating that the audit work was adequately planned and supervised. analyses of capital stock and other owners' equity accounts.

engagement does not include an evaluation of the support for the assumptions underlying the projection. The practitioner's standard report on a compilation of prospective financial statements should include a statement that a compilation is limited in scope and does not enable the practitioner to express an opinion or any other form of assurance on the prospective financial statements or the assumptions. The projection may be included in a document with audited historical financial statements. The practitioner does not have a responsibility to update the projection for future events and circumstances limited to one year. A financial forecast, not a projection, omits all hypothetical assumptions and presents the most likely future financial position.

An accountant who accepts an engagement to compile a financial projection most likely would make the client aware that the: projection may not be included in a document with audited historical financial statements. accountant's responsibility to update the projection for future events and circumstances is limited to one year. projection omits all hypothetical assumptions and presents the most likely future financial position. engagement does not include an evaluation of the support for the assumptions underlying the projection.

find smaller errors. If in an audit engagement, the acceptable levels of both audit risk and materiality are lower, the auditor will plan more work on individual accounts in order to find smaller errors. AU-C 300.02 gives us the strategy of establishing an audit: "Planning an audit involves establishing the overall audit strategy for the engagement and developing an audit plan. Adequate planning benefits the audit of financial statements in several ways, including the following: "Helping the auditor identify and devote appropriate attention to important areas of the audit "Helping the auditor identify and resolve potential problems on a timely basis "Helping the auditor properly organize and manage the audit engagement so that it is performed in an effective and efficient manner..."

As lower acceptable levels of both audit risk and materiality are established, the auditor should plan more work on individual accounts to: find smaller errors. find larger errors. increase the tolerable error in the accounts. decrease the risk of overreliance.

express a qualified or an adverse opinion. A disclosure in the financial statements that is not presented in conformity with an applicable financial reporting framework is an accounting deficiency and requires the issuance of a qualified or an adverse opinion.

If an auditor concludes that a matter involving a risk or an uncertainty is not adequately disclosed in the financial statements in conformity with an applicable financial reporting framework, the auditor should: express a qualified opinion or disclaim an opinion on the financial statements. include an emphasis-of-matter paragraph in his or her report indicating the fact that certain disclosures may not be presented in conformity with GAAP. include an emphasis-of-matter paragraph in his or her report that includes the appropriate information disclosed in conformity with an applicable financial reporting framework. express a qualified or an adverse opinion.

material modifications should be made to conform with the applicable financial reporting framework. The objective of a review of interim financial information of an issuer is to provide the CPA with a basis for reporting whether material modifications should be made to the interim information to comply with the applicable financial reporting framework.

The objective of a review of interim financial information of an issuer is to provide an accountant with a basis for reporting whether: material modifications should be made to conform with the applicable financial reporting framework. a reasonable basis exists for expressing an updated opinion regarding the financial statements that were previously audited. condensed financial statements or pro forma financial information should be included in a registration statement. the financial statements are presented fairly in accordance with the applicable financial reporting framework.

Neither I nor II Statements on Standards for Accounting and Review Services (SSARS) provide standards with respect to compilations and reviews of financial statements. SSARS do not apply to the processing financial data for clients of other CPA firms or to consulting on accounting matters since neither of these would be considered a compilation or a review.

A CPA is required to comply with the provisions of Statements on Standards for Accounting and Review Services when: processing financial data for clients of other CPA firms. consulting on accounting matters. Both I and II I only II only Neither I nor II

"No material weaknesses were identified during the audit." -significant deficiencies implies more than limited assurance

An appropriate statement to make in a written communication of internal control related matters on a financial statement audit of a nonissuer includes: "No significant deficiencies were identified during the audit." "No significant deficiencies or material weaknesses were identified during the audit." "No material weaknesses were identified during the audit." "No internal control deficiencies were identified during the audit."

Request management to further review the impact of likely misstatements that are material either individually or in the aggregate with other misstatements. The auditor should request management to correct all known misstatements, including the effect of prior-period misstatement. The auditor may request management to further review the impact of likely misstatements. If management decides not to correct some or all of the known and likely misstatements communicated to it by the auditor, the auditor should obtain an understanding of management's reasons for not making the corrections and should take that into account when considering the qualitative aspects of the entity's accounting practices.

Bradley CPA identifies both known (factual) and likely (judgmental or projected) misstatements on the financial statement audit of a nonissuer. Which of the following is a required communication with management related to these identified matters? Request management to correct all known misstatements, except for any effect of prior-period misstatement. Request management to correct all known and likely misstatements. Request management to further review the impact of likely misstatements that are material either individually or in the aggregate with other misstatements. Request management to correct only the misstatements that the auditor deems to be material.

cost-benefit considerations in implementing controls. The cost of two reviewers and signers for checks in payment of inventory purchases under $10,000 is probably significantly greater than the internal control benefit.

The average check written by ABC Company for inventory purchases is over $5,000; therefore, the management of ABC Company has decided to require two signatures on their checks for inventory purchases only if the check is in excess of $10,000. This is an example of a reasonable limitation on internal control based on: the small size of the accounting department. compensating controls in the reconciliation of bank statements. cost-benefit considerations in implementing controls. the small number of checks written under $10,000.

Explanatory language in the "Basis for Opinion" section noting that no audit of internal controls over financial reporting is required, although management is required to report on the company's internal control over financial reporting In some circumstances, management is required to report on the company's internal control over financial reporting (ICFR) but such report is not required to be audited and the auditor is not engaged to perform an audit of management's assessment of the effectiveness of ICFR. In such cases, under PCAOB AS 3105.59, the auditor is required to include explanatory language to that effect in the "Basis for Opinion" section. Alternatively, if the auditor issues separate reports on ICFR and the financial statements, under PCAOB AS 2201.88 the required paragraph referencing the separate report should appear in the "Opinion on the Financial Statements" section, immediately following the opinion paragraph. If an auditor is issuing an integrated report, then the reporting requirements of PCAOB AS 2201, An Audit of Internal Control Over Financial Reporting That Is Integrated with an Audit of Financial Statements, should be followed.

Block CPA Firm is engaged to perform a financial statement audit of BIG Company (an issuer). BIG Company is required to report on the company's internal control over financial reporting (ICFR), but that report is not required to be audited. Which of the following would be a required element of the audit report of BIG Company? An opinion on the operating effectiveness of controls in the "Opinion" section A reference to management's separate report on internal control over financial reporting in the "Management's Responsibilities" section Integrating the results of management's report on internal control over financial reporting in the "Opinion" section Explanatory language in the "Basis for Opinion" section noting that no audit of internal controls over financial reporting is required, although management is required to report on the company's internal control over financial reporting

Verify additions and retirements during the year by vouching to supporting documents The question which requires answering is whether the transactions were recorded accurately. In order to test for accuracy of recording, all transactions should be traced to source documents, such as loan agreements. Testing for interest and reviewing minutes (which may not contain detailed balance information) is not effective. "Verify additions and retirements during the year by vouching to supporting documents" will test accuracy.

Which of the following will primarily test for the accuracy of recording of the long-term debt balance outstanding? Trace interest to debt for completeness Recalculate interest expense Verify authorization for borrowing by reading minutes of meetings Verify additions and retirements during the year by vouching to supporting documents

a concurrent opinion on the financial statements taken as a whole. The auditor's report for a performance audit of a governmental entity in accordance with Government Auditing Standards should contain: the objectives, scope, and methodology of the audit, the audit results, including findings, conclusions, and recommendations, as appropriate, a reference to compliance with generally accepted government auditing standards, the views of responsible officials, and if applicable, the nature of any privileged and confidential information omitted. A concurrent opinion on the historical financial statements is not the objective of the performance audit and is not required.

An auditor was engaged to conduct a performance audit of a governmental entity in accordance with Government Auditing Standards. These standards do not require, as part of this auditor's report: the objectives, scope, and methodology of the audit. the audit results. the nature of any privileged and confidential information omitted. a concurrent opinion on the financial statements taken as a whole.

risk assessment. An entity's risk assessment process for financial reporting purposes is its identification, analysis, and management of risks relevant to the preparation of financial statements that are fairly presented in conformity with the entity's applicable reporting framework.

The component of internal control which relates to the entity's ability to identify, analyze, and decide how to respond to risks relevant to the accounting and financial reporting process is: the control environment. risk assessment. control activities. monitoring.

issue an accountant's review report with a separate paragraph discussing the change in engagement scope. When an accountant determines that a change in the scope of an engagement from an audit to a review is appropriate, the accountant would issue a review report and would not refer to the original engagement, any procedures performed as part of the engagement, or any scope limitation. The accountant should consider the additional audit effort and cost required to complete the audit, evaluate the possibility that financial statement information affected by the limitation on work to be performed may be incorrect or incomplete, and consider the reason given for the client's request and assess whether the request is reasonable.

A CPA is engaged to audit the financial statements of a nonissuer. After the audit begins, the client's management questions the extent of procedures and objects to the confirmation of certain contracts. The client asks the accountant to change the scope of the engagement from an audit to a review. Under these circumstances, the accountant should do each of the following, except: issue an accountant's review report with a separate paragraph discussing the change in engagement scope. consider the additional audit effort and cost required to complete the audit. evaluate the possibility that financial statement information affected by the limitation on work to be performed may be incorrect or incomplete. consider the reason given for the client's request and assess whether the request is reasonable.

there will usually be differences between the projected and actual results because events and circumstances frequently do not occur as expected. The practitioner's standard report on a compilation on a financial projection should include the following: An identification of the prospective financial statements presented by the responsible party A statement that the practitioner has compiled the prospective financial statements in accordance with standards established by the American Institute of Certified Public Accountants A statement that a compilation is limited in scope and does not enable the practitioner to express an opinion or any other form of assurance on the prospective financial statements or the assumptions A caveat that the prospective results may not be achieved A statement that the practitioner assumes no responsibility to update the report for events and circumstances occurring after the date of the report A statement describing the special purpose for which the projection was prepared A separate paragraph that restricts the use of the report because it is intended to be used solely by the specified parties The manual or printed signature of the practitioner's firm The date of the compilation report If the projection does not contain a range, the report should also include:a statement that there will usually be differences between the projected and actual results, because events and circumstances frequently do not occur as expected, and those differences may be material.a statement that the practitioner has no responsibility to update the report for events and circumstances occurring after the date of the report.

An accountant's compilation report on a financial projection that does not contain a range should include a statement that: there will usually be differences between the projected and actual results because events and circumstances frequently do not occur as expected. the compilation includes an evaluation of the support for the assumptions underlying the projection. the accountant's responsibility to update the report for future events and circumstances is limited to one year. the projection is limited to presenting, in the form of financial statements, information that is the accountant's representation.

Review the client's control procedures over the safeguarding of inventory, and perform a physical inventory count on the last day of the current year. The auditor should plan and perform substantive procedures to be responsive to the related assessment of a risk of material misstatement. Below is a partial list of audit procedures: Inspection of records or documents (internal or external, in paper form, electronic form, or other media) Physical inspection of tangible assets Observation (looking at a process or procedure being performed by others) Inquiry (seeking information of knowledgeable persons, both financial and nonfinancial, inside or outside the entity) Recalculation (verifying the mathematical accuracy of documents or records) Specifically, as of the balance sheet date, the auditor should observe the inventory count and, through tests and inquiries, be satisfied with the effectiveness of the methods of inventory taking and the measure of reliance which may be placed upon the client's representations about the quantities and physical condition of the inventories. The observation should be coupled with inspection of the records of any client's counts and procedures relating to the physical inventory on which the balance sheet inventory is based. Merely reviewing client control procedures, applying analytical procedures, and performing counts throughout the year is not sufficient to adequately address the risk of a material misstatement; the inventory must be physically verified at year-end.

An audit team has concluded that inventory is highly susceptible to misappropriation and that a potential misstatement would be material to the financial statements. How should the audit team address the audit procedures to the increased risk? Review the client's control procedures over the safeguarding of inventory, and perform a physical inventory count on the last day of the current year. Review the client's control procedures over the safeguarding of inventory, incorporate the use of substantive analytical procedures, and develop an expectation. Review the client's control procedures over the safeguarding of inventory, but do not modify substantive procedures over inventory. Review the client's control procedures over the safeguarding of inventory, and perform physical inventory counts throughout the current year.

there is an unacceptably high risk that the actual misstatements in the population exceed the tolerable misstatement because the total projected misstatement is more than the tolerable misstatement. The auditor projects the sample results to the entire population. In this question, an overstatement of $3,700 and an understatement of $200 were found when the auditor sampled 5% of the total population. Projecting the misstatement to the entire population results in $74,000 in overstatements and $4,000 for understatements. Since the total projected misstatement is more than the tolerable misstatement, there is an unacceptably high risk that the actual misstatements in the population exceed the tolerable misstatement. 5% of total population = 1/20 of total population $3,700 overstatement × 20 = $74,000 $200 understatement × 20 = $4,000 ($74,000 overstatement - 4,000 understatement) - $60,000 = $10,000 Therefore, the projected misstatement exceeds the tolerable misstatement by $10,000. Another way to calculate the excess overstatement over the tolerable misstatement is by netting the $3,700 overstatement and $200 understatement. $3,700 overstatement - $200 understatement = $3,500 net overstatement in the sample $3,500 × 20 = $70,000 projected overstatement $70,000 - $60,000 = $10,000 excess

An auditor established a $60,000 tolerable misstatement for an asset with an account balance of $1 million. The auditor selected a sample of every twentieth item from the population that represented the asset account balance and discovered overstatements of $3,700 and understatements of $200. Under these circumstances, the auditor most likely would conclude that: there is an unacceptably high risk that the actual misstatements in the population exceed the tolerable misstatement because the total projected misstatement is more than the tolerable misstatement. there is an unacceptably high risk that the tolerable misstatement exceeds the sum of actual overstatements and understatements. the asset account is fairly stated because the total projected misstatement is less than the tolerable misstatement. the asset account is fairly stated because the tolerable misstatement exceeds the net of projected actual overstatements and understatements.

the auditor's responsibility for ensuring that the appropriate level of management is aware of any significant deficiencies that come to the auditor's attention. The engagement letter should contain information such as: the objective of the audit (an expression of an opinion on the financial statements); the fact that management is responsible for:the financial statements,establishing and maintaining effective internal control over financial reporting,identifying and ensuring that the entity complies with laws and regulations,adjusting the financial statements to correct material misstatements,making all financial records and related information available to the auditor, andproviding the auditor with a letter that confirms certain representations made during the audit; the scope of the audit work to be performed (in accordance with GAAS); the fact that the purpose of the audit is not to detect fraud but to enable the auditor to express an opinion as to the fairness of the financial statements; mention that an audit includes obtaining an understanding of internal control and that the audit committee will be made aware of any discovered significant deficiencies; additional work to be performed, such as tax, consulting, or other services (if applicable); any limitations or restrictions on the scope of the study; work to be performed by the client's staff (if applicable); the basis of the auditor's fee; and the audit work schedule and estimated date of completion. This list is not inclusive, but it is illustrative of items that should be present. Items that would not be addressed in an engagement letter would be the auditor's responsibility for determining the preliminary judgments about materiality and audit risk factors, management's responsibility for identifying mitigating factors when the auditor has doubt about the entity's ability to continue as a going concern, or management's responsibility for providing the auditor with an assessment of the risk of material misstatement due to fraud (this is the auditor's responsibility).

An auditor is required to establish an understanding with a client regarding the services to be performed for each engagement. This understanding generally includes: the auditor's responsibility for ensuring that the appropriate level of management is aware of any significant deficiencies that come to the auditor's attention. management's responsibility for identifying mitigating factors when the auditor has doubt about the entity's ability to continue as a going concern. the auditor's responsibility for determining preliminary judgments about materiality and audit risk factors. management's responsibility for providing the auditor with an assessment of the risk of material misstatement due to fraud.

limited to the specific event referenced. An auditor dual dates an audit report to limit the responsibility taken for subsequent events to the specific event referenced. By dual dating the report for a subsequent event that has occurred after the original date of the auditor's report, the auditor has chosen not to extend his responsibility (and procedures) either to all subsequent events occurring through the date of the report or to all events occurring since the original date of the auditor's report. The dual dating of the audit report references the note to the financial statements which discloses the specific subsequent event that has occurred after the original date of the auditor's report. When dual dating is not used, the auditor's responsibility is extended to include subsequent events occurring up to the date of the audit report.

An auditor issued an audit report that was dual dated for a subsequent event occurring after the original date of the auditor's report but before issuance of the related financial statements. The auditor's responsibility for events occurring subsequent to the original report date was: limited to include only events occurring up to the date of the last subsequent event referenced. limited to the specific event referenced. extended to subsequent events occurring through the date of the report. extended to include all events occurring since the original date of the auditor's report.

indicates whether the information is fairly stated in all material respects in relation to the complete financial statements. The audit report on condensed financial statements should state, "The information set forth in the accompanying condensed consolidated financial statements is fairly stated in all material respects to the consolidated financial statements from which it has been derived."

An auditor may report on condensed financial statements that are derived from a complete set of audited financial statements only if the auditor: expresses an unmodified opinion on the audited financial statements from which the condensed financial statements are derived. indicates whether the information is fairly stated in all material respects in relation to the complete financial statements. determines that the condensed financial statements include all the disclosures necessary for the complete set of financial statements. presents the condensed financial statements in comparative form with the prior year's condensed financial statements.

neither an unjustified accounting change nor a material weakness in internal control. The auditor is required to state whether GAAP has been consistently applied. Inconsistency in the application of GAAP is a common reason for qualified opinions by auditors. Therefore, an unjustified accounting change would not result in an unmodified opinion with an emphasis-of-matter paragraph. Auditors are required to communicate material weaknesses in internal control to both company management and those charged with governance. However, this is done in a separate communication and is not included as an emphasis-of-matter paragraph to an unmodified opinion in the auditor's report. Emphasis-of-matter paragraphs are included in the auditor's report when a matter appropriately presented or disclosed in the financial statements exists that, in the auditor's professional judgment, is of such importance that it is fundamental to the users' understanding of the financial statements.

An auditor would express an unmodified opinion with an emphasis-of-matter paragraph added to the auditor's report of a nonissuer for: an unjustified accounting change. a material weakness in internal control. both an unjustified accounting change and a material weakness in internal control. neither an unjustified accounting change nor a material weakness in internal control.

The accuracy of significant judgments to be used in the preparation of the financial statements An engagement letter to prepare financial statements on behalf of management should state that management is responsible for the accuracy of significant judgments to be used, among other items such as management's responsibility for the detection of fraud. Financial statement disclosures are the responsibility of the accountant, not management. Neither party discloses the framework on the face of the financial statements. No assurance is provided over financial statement preparation engagements. Helpful Hint: When you see "level of assurance," you should always think accountant or auditor, not management.

For an engagement to prepare financial statements, which of the following items should be identified in the engagement letter as management's responsibility? The preparation of financial statement disclosures The determination of the level of assurance over the financial statements The accuracy of significant judgments to be used in the preparation of the financial statements The inclusion of the financial reporting framework description on the face of the financial statements

When issuing an opinion or a disclaimer of on the financial statements When providing an opinion or a disclaimer on government financial statements after performing an audit under GAGAS and GAAS standards, the auditor should also report on internal control over financial reporting and on compliance with laws, regulations, and provisions of contracts or grant agreements.

For financial statement audits performed in accordance with generally accepted government auditing standards (GAGAS) in addition to GAAS (generally accepted auditing standards), when also must the auditor issue a report on internal control over financial reporting and on compliance with laws, regulations, and provisions of contracts or grant agreements? When engaged by the reporting entity to issue such reports When issuing an opinion on the financial statements When issuing an opinion or a disclaimer of opinion on the financial statements When issuing a qualified or adverse opinion on the financial statements

Identifying entries posted to incorrect accounts Attribute sampling is used for tests of controls. This type of sampling answers the question of "how many," and the auditor would be looking for the appearance (or absence) of a specific characteristic. To estimate amounts, evaluate the reasonableness of a number in the financial statements, or select a sample of receivables for confirmation, the auditor would use substantive procedures. The only answer choice dealing with tests of controls is "identifying entries posted to incorrect accounts."

For which of the following audit tests would a CPA most likely use attribute sampling? Identifying entries posted to incorrect accounts Estimating the amount in an expense account Evaluating the reasonableness of depreciation expense Selecting receivables for confirmation of account balances

perform the planned auditing procedures closer to the balance sheet date. When an auditor lowers the amount of tolerable misstatement, a more careful audit is planned to detect small misstatements. Of the items listed above, only "perform the planned auditing procedures closer to the balance sheet date" results in a more careful audit. The other choices result in a less careful audit.

Holding other planning considerations equal, a decrease in the amount of misstatements in a class of transactions that an auditor could tolerate most likely would cause the auditor to: apply the planned substantive tests prior to the balance sheet date. perform the planned auditing procedures closer to the balance sheet date. increase the assessed level of control risk for relevant financial statement assertions. decrease the extent of auditing procedures to be applied to the class of transactions.

beta risk; effectiveness The risk of incorrect acceptance and the risk of assessing control risk too low (beta risk) relate to the effectiveness of the audit in detecting existing material misstatement. The risk of incorrect rejection and the risk of assessing control risk too high (alpha risk) relate to the efficiency of the audit.

In testing controls for operating effectiveness, the risk of assessing control risk too low is called _______, which impacts __________. beta risk; efficiency alpha risk; efficiency alpha risk; effectiveness beta risk; effectiveness

Scope: Different; Procedures: Similar; Objective: Different Obtaining an understanding of an entity's controls is not sufficient to test their operating effectiveness, unless some automation provides for the consistent operation of the controls. Obtaining audit evidence about the implementation of a manual control at a point in time does not provide audit evidence about the operating effectiveness of the control at other times during the period. Testing the operating effectiveness of a control means determining if the control is operating as designed and whether the person performing the control possesses the necessary authority and competence to perform the control effectively. Therefore, the design and operating effectiveness of an entity's internal control over financial reporting compared to obtaining an understanding of internal control and assessing control risk as part of an audit have similar procedures; however, they have different scopes and very different objectives.

How do the scope, procedures, and objective of an engagement to audit the design and operating effectiveness of an entity's internal control over financial reporting compare to those for obtaining an understanding of internal control and assessing control risk as part of an audit? Scope: Similar; Procedures: Different; Objective: Similar Scope: Different; Procedures: Different; Objective: Different Scope: Different; Procedures: Similar; Objective: Different Scope: Different; Procedures: Similar; Objective: Similar

Review the risk assessment of the opening balances of the financial statements. The successor auditor must communicate with the predecessor auditor before accepting an engagement, because the predecessor may provide information regarding disagreements about important accounting and auditing matters that will bear on the decision of whether or not to accept the engagement. Once an engagement has been accepted, the successor may make specific inquiries of the predecessor as to matters that affect the conduct of the audit, and review the predecessor's workpapers, including documentation of planning, internal control, audit results, and other matters of continuing accounting and auditing significance, such as the working paper analysis of balance sheet accounts, and those relating to contingencies. The successor auditor's review of the predecessor's workpapers may affect the nature, extent, and timing of the successor's procedures with respect to the opening balances and consistency of accounting principles.

If the predecessor auditor refuses to give the current auditor of a nonissuer access to the documentation, what should the current auditor do? Review the risk assessment of the opening balances of the financial statements. Withdraw from the engagement. Disclaim an opinion due to a scope limitation. Discuss the matter with the client's legal counsel.

Opinion paragraph The auditor's standard report of a nonissuer identifies the financial statements audited in an opening (opinion) paragraph, which identifies the applicable financial reporting framework and accomplishes the following: Identifies the entity whose financial statements have been audited States that the financial statements have been audited Identifies the title of each statement that the financial statements comprise Refers to the notes Specifies the dates of or periods covered by each financial statement that the financial statements comprise

In which of the following paragraphs of an auditor's report of a nonissuer does an auditor communicate the specific financial statements covered by the audit? Auditor's responsibility paragraph Basis-for-opinion paragraph Opinion paragraph Emphasis-of-matter or other-matter paragraph

The group audit report discloses that the group auditor took responsibility for evaluating the adjustment made to convert the component's financial statements to the same framework used by the group. The determination of whether to make reference to a component auditor in the auditor's report on the group financial statements requires, when reference is made to a component auditor's report on financial statements prepared using a different financial reporting framework, the auditor's report on the group financial statements to disclose that the auditor of the group financial statements is taking responsibility for evaluating the appropriateness of the adjustments to convert the component unit's financial statements to the financial reporting framework used by the group.

In a group financial statement audit, the group audit report may make reference to a component auditor's report in which of the following circumstances when the financial statements of the component unit are prepared using an applicable financial reporting framework different than the group? The component audit report discloses that the component auditor took responsibility for evaluating the adjustment made to convert the component's financial statements to the same framework used by the group. The group audit report discloses that the group auditor took responsibility for evaluating the adjustment made to convert the component's financial statements to the same framework used by the group. Reference to a component auditor's report is prohibited in this circumstance. The group audit report discloses that the component auditor took responsibility for evaluating the adjustment made to convert the component's financial statements to the same framework used by the group.

Recalculating benefits for selected participants based on the plan provisions using relevant service and salary history to support the recorded benefits paid to the participants Defined benefit plans are commonly referred to as pension plans. They afford the beneficiary a fixed benefit in retirement based upon the plan agreement. Such payments typically are based on the beneficiary's age, salary, length of service, and a set rate per these factors. Given the payments are set contractually, the auditor should recalculate the expected payment amount and compare it to the actual payment amount. Inquiry of the plan administrator is an important internal control evidence-gathering activity, but it is not sufficient to test the actual payments. Sending confirmations confirms receipt of the payment, but it does not confirm whether the payment was made in the accurate amount per the plan documents. Comparing the benefits recorded in the general ledger (GL) to the payments does not test the accuracy of such payments. The client could have calculated and paid the wrong amount, but they would still agree in the GL.

In an audit of a nonissuer's defined benefit pension plan, which of the following procedures most effectively tests that benefit payments to plan participants are paid in accordance with the plan document? Recalculating benefits for selected participants based on the plan provisions using relevant service and salary history to support the recorded benefits paid to the participants Inquiring of the plan administrator about the internal control procedures in place to ensure that the benefit payments to plan participants are paid in accordance with the plan provisions Sending confirmations to plan participants to ascertain that the amount paid to them in the fiscal year is the same as recorded in the general ledger account related to benefit payments Testing the accuracy and completeness of benefit payments made to plan participants and comparing the benefit payments recorded in the general ledger to the benefit payment records in participant records

the combined assessed level of inherent risk and control risk relative to accounts receivable is low. The purpose of a confirmation is to help the auditor verify client financial statement assertions. Confirmation requests can be prepared in two ways. With a positive confirmation, third parties are either asked to fill in amounts from their records regarding transactions with the audit client, or requested to indicate whether they agree with information already stated in the confirmation. The positive type of confirmation is a very strong form of audit evidence. With a negative confirmation, the third party usually responds only when the information provided by the auditor from the client's records does not agree with the third party's records. Negative confirmations are considered to be a weak form of evidence. Therefore, the answer, "The combined assessed level of inherent risk and control risk relative to accounts receivable is low," is correct; the auditor is not relying heavily on the negative confirmation evidence due to the low levels of inherent risk and control risk already measured in the audit process.

In auditing accounts receivable, the negative form of confirmation request most likely would be used when: the total recorded amount of accounts receivable is immaterial to the financial statements taken as a whole. response rates to properly designed positive confirmation requests were inadequate in prior years. recipients are likely to return positive confirmation requests without verifying the accuracy of the information. the combined assessed level of inherent risk and control risk relative to accounts receivable is low.

The financial statements fail to disclose information that is required by generally accepted accounting principles. If the financial statements, including accompanying notes, fail to disclose information that is required by GAAP, the auditor should express a qualified or adverse opinion. The choice of opinion (qualified or adverse) would depend on the pervasiveness of the GAAP departure. Material and pervasive omitted disclosures would likely lead to an adverse opinion, while material but not pervasive omitted disclosures would likely to lead to a qualified opinion. A company can receive a going concern opinion while still having an unqualified opinion on the financial statements as a whole. Scope limitations, such as the inability to observe inventory, would lead the auditor to select between a qualified opinion or disclaimer of opinion.

In which of the following situations would an auditor ordinarily choose between expressing a qualified opinion or an adverse opinion? The auditor did not observe the entity's physical inventory and is unable to become satisfied as to its balance by other auditing procedures. The financial statements fail to disclose information that is required by generally accepted accounting principles. The auditor is asked to report only on the entity's balance sheet and not on the other basic financial statements. Events disclosed in the financial statements cause the auditor to have substantial doubt about the entity's ability to continue as a going concern.

seek information about contingent liabilities and security agreements. One portion of the standard bank confirmation form requests the bank to list all known contingent liabilities and security agreements existing between the bank and auditor's client. Such information can only be obtained from bank confirmations. The alternatives can all be achieved by other procedures.

One of the purposes for sending a confirmation request to all banks with which the client has done business during the year under audit is to: provide the data necessary to prepare a proof of cash. request a cutoff bank statement and related checks be sent to the auditor. detect kiting activities that may otherwise not be discovered. seek information about contingent liabilities and security agreements.

II only If two or more auditors examine a portion of a client's financial statements, the decision must be made as to who the principal auditor is. Normally the auditor who has examined the major portion of the financial statements is the principal auditor, and he or she will issue the audit opinion. If the principal auditor is able to satisfy himself about the quality of another auditor's work and is willing to take responsibility for that work, no mention of the other auditor is necessary. The type of opinion (unmodified, qualified, adverse, or disclaimer of opinion) is not relevant. However, if the principal auditor is unable to satisfy himself about the quality of the other auditor's work, reference quality of other auditor is made and a clear separation of responsibilities is described in the audit opinion.

Pell, CPA, decides to serve as principal auditor in the audit of the financial statements of Tech Consolidated, Inc. Smith, CPA, audits one of Tech's subsidiaries. In which situation(s) should Pell make reference to Smith's audit? I. Pell reviews Smith's workpapers and assumes responsibility for Smith's work, but expresses a qualified opinion on Tech's financial statements. II. Pell is unable to review Smith's workpapers; however, Pell's inquiries indicated that Smith has an excellent reputation for professional competence and integrity. I only II only Both I and II Neither I nor II

2.0 times First, it is necessary to compute ending inventory: Beginning inventory $ 6,000 Purchases 24,000 Goods available 30,000 Less cost of goods sold 18,000 Ending inventory $12,000 Inventory Turnover = COGS/Avg. Inventory =18,000 / ((6,000+12,000) X 0.5) =18,000/9,000 =2.0 TIMES

Selected data pertaining to Lore Co. for the calendar year 20X1 is as follows: Net cash sales $ 3,000Cost of goods sold 18,000Inventory at beginning of year 6,000Purchases 24,000Accounts receivable at beginningof year 20,000Accounts receivable at end of year 22,000 What was the inventory turnover for 20X1? 1.2 times 1.5 times 2.0 times 3.0 times

The auditor should perform limited procedures Supplementary information is defined by AU-C 725 as "information presented outside the basic financial statements, excluding required supplementary information that is not considered necessary for the financial statements to be fairly presented in accordance with the applicable financial reporting framework. Such information may be presented in a document containing the audited financial statements or separate from the financial statements." The auditor should perform limited procedures, including the following: Inquire of management about the purpose of the supplementary information, the criteria used by management to prepare the information, and any significant assumptions or interpretations underlying the measurement or presentation of the information Compare and reconcile the information to the underlying accounting and other records used in preparing the financial statements Inquire of management about any significant assumptions or interpretations Perform procedures to test the completeness and accuracy of the information Obtain written representations from management regarding the information The auditor should report on supplementary information either in an other-matter paragraph in the auditor's report on the financial statements or in a separate report on the supplementary information.

The client's financial reporting includes supplementary financial information outside the basic financial statements but required by the Financial Accounting Standards Board (FASB). Which of the following statements is correct regarding the auditor's responsibility for this supplementary financial information? The auditor should perform limited procedures. The auditor is not required to report omissions. The auditor should read the supplementary financial information. The auditor should apply tests of details of transactions.

None of the answer choices are correct. Be careful of the word "only." Observation and inspection are ways to support inquiries (interrogation) of management, but other empirical evidence may be available. Risk assessment procedures include: inquiries of management and others within the entity, analytical procedures, and observation and inspection. In addition, the auditor may perform other procedures where the information obtained may be helpful in identifying risks of material misstatement. These procedures may include inquiries of others outside the entity (such as legal counsel), reviewing information from external sources (such as reports by analysts or banks), or reviewing trade journals or regulatory publications.

The only way to determine risk within an organization is through: observation and inspection. inspection and interrogation. observation, inspection, and interrogation. None of the answer choices are correct.

the Office of Management and Budget (OMB). The Director of the Office of Management and Budget (OMB) has the authority to develop government-wide guidelines and policy on performing audits to comply with the Single Audit Act. The Government Accountability Office (GAO) establishes standards for performing audits in conformance with generally accepted government auditing standards, which is distinct from the Single Audit Act.

The regulatory body that has the primary authority to develop requirements for performing federal program compliance audits in accordance with the Single Audit Act is: the Office of Management and Budget (OMB). the Government Accountability Office (GAO). the Public Company Accounting Oversight Board (PCAOB). Congress.

a disclaimer of opinion about whether all questioned costs have been reported. An auditor reporting in accordance with the Uniform Guidance rules on a schedule of findings and questioned costs would not include a disclaimer of opinion as to whether all questioned costs have been reported. Such reporting would significantly alter the purpose of this portion of the auditor's report.

The schedule of findings and questioned costs, included in an auditor's report under the Uniform Guidance rules for nonprofit organizations expending federal awards, would include all of the following, except: significant deficiencies. material noncompliance with the provisions of laws, regulations, contracts, or grant agreements. known questioned costs when likely questioned costs are greater than $25,000 for a type of compliance requirement for a major program. a disclaimer of opinion about whether all questioned costs have been reported.

The objective of a compilation is to assist management in presenting financial information in the form of financial statements. The phrase "the objective of a compilation is to assist management in presenting financial information in the form of financial statements" is the only phrase in the answer choices found in a compilation report (AR-C 80.04). The phrases "substantially less in scope that an audit" and "limited assurance" are found in a review report (AR-C 90.07). Audit and attestation engagements are performing in accordance with standards established by the Auditing Standards Board.

The standard compilation report includes which statement or phrase? A compilation is substantially less in scope than a review or an audit. The accountant does not express an opinion but expresses only limited assurance on the compiled financial statements. The objective of a compilation is to assist management in presenting financial information in the form of financial statements. The accountant has compiled the financial statements in accordance with standards established by the Auditing Standards Board.

the accountant should verify the completeness of information provided by management for the financial statements. The answer choice "the accountant should verify the completeness of information provided by management for the financial statements" is correct. When an accountant prepares financial statements, this is known as a compilation. The accountant will take financial data provided by management at face value. In other words, no effort is undertaken to verify the completeness of the information or provide any assurance that there are material modifications needed for the financial statements to be prepared in accordance with an applicable financial reporting framework. The remaining answer choices (preparing an engagement letter, determining the applicable financial framework, and indicating no assurance is provided) are all appropriate actions to undertake when an accountant is engaged to prepare financial statements.

When an accountant is engaged to prepare financial statements, each of the following requirements applies, except: the accountant should verify the completeness of information provided by management for the financial statements. the engagement documentation should include the engagement letter and a copy of the prepared financial statements. the agreed-upon terms of the engagement should include identification of the applicable financial framework to be used. the accountant should include a statement on each page of the financial statements indicating that no assurance is provided.

whether there has been any turnover of the component's board members. When the auditor of a parent entity is also the auditor of a component unit, the factors that may influence the decision whether to obtain a separate audit engagement letter from the component include: who engages the component auditor; whether a separate auditor's report is to be issued on the component; legal requirements regarding the appointment of the auditor; the degree of ownership by parent; and the degree of independence of the component management from the parent entity. Whether there has been any turnover of the component's board members is not a factor that would ordinarily influence the decision to obtain a separate engagement letter from the component.

When an auditor of a parent nonissuer is also the auditor of a component unit, then each of the following factors would ordinarily influence the decision to obtain a separate engagement letter from the component, except: the legal requirements regarding the appointment of the auditor. whether a separate audit report is to be issued on the component. whether there has been any turnover of the component's board members. the degree of independence of the component management from the parent entity.

all inventory owned by the client is on hand at the time of the count.

When auditing inventories, an auditor would least likely verify that: the financial statement presentation of inventories is appropriate. damaged goods and obsolete items have been properly accounted for. all inventory owned by the client is on hand at the time of the count. the client has used proper inventory pricing.

understand the accounting principles of the industry in which the entity operates. This problem describes a review of financial statements. One of the many steps a CPA should take when engaged to review financial statements of a nonissuer is the CPA should acquire the necessary knowledge of the client's industry accounting principles and practices. A review does not involve understanding the client's internal control (which would be involved in assessing the control risk or the risk of material misstatement), assessing fraud risk, testing accounting records, or other procedures normally performed in an audit (such as confirmations with the lawyer or developing audit programs).

When providing limited assurance that the financial statements of a nonissuer require no material modifications to be in accordance with generally accepted accounting principles, the accountant should: assess the risk that a material misstatement could occur in a financial statement assertion. confirm with the entity's lawyer that material loss contingencies are disclosed. understand the accounting principles of the industry in which the entity operates. develop audit programs to determine whether the entity's financial statements are fairly presented.

the same as the original report. AU-C 560.16 specifies that if management revises the financial statements, and the auditor reissues a report of the financial statements, the date of the reissued report should be the same as the original report. If the auditor's opinion on the revised financial statements differs from the opinion the auditor previously expressed, the auditor should disclose the following matters in an emphasis-of-matter or other-matter paragraph: The date of the auditor's previous report The type of opinion previously expressed The substantive reasons for the different opinion That the auditor's opinion on the revised financial statements is different from the auditor's previous opinion Use of the original report date in a reissued report removes any implication that records, transactions, or events after that date have been examined or reviewed.

When the auditor reissues a report of the financial statements, the date of the reissued report should be: changed to the date the report was reissued. both the original date and the date the report was reissued. changed to the date of the most recently released report. the same as the original report.

obtain a representation letter from the successor auditor. When the auditor reissues a report of the financial statements, the auditor should read the financial statements of the subsequent period, compare the prior-period financial statements to the subsequent period, request a written representation from management, and obtain a representation letter from the successor auditor. Review procedures are not performed.

When the auditor reissues a report of the financial statements, the responsibility of the auditor with respect to the reissued report is which of the following? The auditor: should adjust the financial statements as needed or qualify the opinion. must disclaim an opinion. should perform limited review procedures as necessary. obtain a representation letter from the successor auditor.

vendors with whom the entity has previously done business. The key to this type of question is to ask: Which set of items will indicate whether every amount that should be recorded has been? Is the selected item, which should be recorded, actually recorded? Should this selected item, which is not recorded, actually be recorded? In this case, the questions concern the completeness assertion, which states that all amounts are included and none is omitted. With respect to the completeness of accounts payable, you should ask: Which population will indicate where all accounts payable are recorded? Does the auditee owe some amount to anyone else? Thus, the auditor must select from a population containing items that are not included but which possibly should be. For accounts payable, confirmation with vendors with whom the entity has previously done business, but to whom no recorded payable balance is due, is the appropriate population ("If the auditee asks this vendor if any amount is owed, will the answer be zero?") Responses from a vendor to whom the client does owe a payable would reveal an unrecorded (i.e., incomplete) liability. Keep in mind that the question asks about populations appropriate for confirmation for completeness. The other three populations listed are appropriate for other procedures; mere comparison of amounts recorded in the accounts payable subsidiary ledger to payees of checks drawn in the month after the year-end or to invoices filed in the entity's open invoice file may reveal an unrecorded liability. However, confirmation of a recorded amount is unlikely to provide any additional evidence about completeness.

When using confirmations to provide evidence about the completeness assertion for accounts payable, the appropriate population most likely would be: vendors with whom the entity has previously done business. amounts recorded in the accounts payable subsidiary ledger. payees of checks drawn in the month after the year-end. invoices filed in the entity's open invoice file.

Perform the engagement even though independence is compromised A compilation does not involve performing inquiry or analytical procedures, obtaining an understanding of the entity's internal control, assessing fraud risk, or testing accounting records. The objective of a compilation is to assist management in presenting financial information in the form of financial statements without undertaking to obtain or provide any assurance that there are no material modifications that should be made to the financial statements in order for the statements to be in conformity with the applicable financial reporting framework. The accountant is permitted to perform a compilation when not independent; the report must disclose the lack of independence.

Which of the following actions could an accountant take when engaged to compile a company's financial statements in accordance with Statements on Standards for Accounting and Review Services (SSARS)? Perform analytical procedures Express negative assurance on the financial statements Make management inquiries and examine internal controls Perform the engagement even though independence is compromised

The client's willingness to issue revised financial statements or other disclosures to persons known to be relying on the financial statement The auditor would consider it most important that the client willingly issue revised financial statements or other disclosures to persons known to be relying on the financial statement. A subsequent discovery is the discovery of facts that existed at the date of the audit report but that were not known at that time. The auditor has no obligation to perform any further audit procedures after the report date, but new information may come to the auditor's attention. If this happens, the auditor should consider consulting with an attorney and determine if the subsequently discovered information (1) is reliable, (2) existed at the date of the report, (3) is material to the report (would or might change opinion), and (4) is applicable to the report that is still being relied on. If all four conditions exist, the auditor should: advise the client to revise and reissue the statements and auditor's report. make disclosure in financial statements of a subsequent period if the date of issue is imminent. notify persons relying on statements of the facts that are known to the auditor, if the effect cannot be promptly determined and it appears the financial statements will be revised after investigation. discuss with the SEC or other regulatory body, if appropriate. If the client refuses to cooperate in disclosing the facts, the auditor should notify the client (management and board of directors) that the auditor's report cannot be associated with statements, regulatory bodies, and each person known by the auditor to be relying on the statements.

Which of the following factors should an auditor consider most important upon subsequent discovery of facts that existed at the date of the audit report and would have affected the report? The cost-to-benefit ratio of performing additional procedures to better determine the impact of the newly discovered facts The client's willingness to issue revised financial statements or other disclosures to persons known to be relying on the financial statement The potential impact on financial statements and associated audit reports for the previous five years The client's willingness to pay additional fees for the additional procedures to be performed

Assigning personnel to engagements The elements of a CPA firm's quality control system are identified in Statement on Quality Control Standards (SQCS) 8 as: leadership responsibilities for quality within the firm, relevant ethical requirements, acceptance and continuance of client relationships and specific engagements, human resources, engagement performance, and monitoring. Policies and procedures for assigning personnel to engagements ensure that only technically trained and proficient personnel perform the audit work. Audit risk, materiality, and statistical sampling techniques are considered in the planning and performance of an audit of financial statements. Compliance with laws and regulations is an audit objective.

Which of the following is an element of a CPA firm's quality control system that should be considered in establishing its quality control policies and procedures? Complying with laws and regulations Using statistical sampling techniques Assigning personnel to engagements Considering audit risk and materiality

The firm is able to complete the engagement within the firm's time budget. Matters to consider in accepting or continuing the client engagement include whether: firm personnel have experience with relevant industries or underlying subject matter or the ability to effectively gain the necessary knowledge; firm personnel have experience with relevant regulatory or reporting requirements, or the ability to effectively gain the necessary competencies; the firm has sufficient personnel with the necessary competence and capabilities; specialists are available, if needed; individuals meeting the criteria and eligibility requirements to perform an engagement quality control review are available, where applicable; and the firm is able to complete the engagement within the reporting deadline.

Which of the following is not a consideration in accepting or continuing a compilation or review engagement? The firm has sufficient personnel with the necessary capabilities and competencies. Specialists are available, if needed. The firm is able to complete the engagement within the reporting deadline. The firm is able to complete the engagement within the firm's time budget.

Controls over dual authorization for all check disbursements Entity-level controls include, but are not limited to: controls related to the control environment, controls over management override, the company's risk assessment process, centralized processing and controls, including shared service environments, controls to monitor results of operations, controls to monitor other controls, including activities to monitor the internal audit function, the audit committee, and self-assessment programs, controls over the period-end financial reporting process, and policies that address significant business control and risk management practices.

Which of the following is not considered to be an entity-level control within the entity's internal controls over financial reporting? Controls over management override Controls to monitor results of operations Controls over dual authorization for all check disbursements Controls over the period-end financial reporting process

Assessing control risk below the maximum is required in order to have a reasonable basis for extending audit conclusions from an interim date to the balance sheet date. Assessing control risk below the maximum is not required to have a reasonable basis for extending audit conclusions from an interim date to the balance sheet date. However, if the auditor assesses control risk at the maximum during the remaining period, he or she should consider whether the effectiveness of certain of the substantive tests to cover that period will be impaired.

Which of the following is not generally considered necessary if an auditor applies principal substantive tests to details of balance sheet accounts at an interim date? The auditor should assess the difficulty in controlling the incremental audit risk associated with applying substantive tests at an interim date. The auditor should consider whether the year-end balances of the particular asset or liability accounts that might be selected for interim examination are reasonably predictable with respect to amount, relative significance, and composition. The auditor should consider whether there are rapidly changing business conditions or circumstances that might predispose management to misstate financial statements in the remaining period. Assessing control risk below the maximum is required in order to have a reasonable basis for extending audit conclusions from an interim date to the balance sheet date.

Obtaining confirmation of inventories that are pledged under loan agreements "Obtaining confirmation of inventories that are pledged under loan agreements" is correct. Presentation and disclosure is one of the essential assertions management explicitly makes when organizing financial statements, namely that accounts are properly categorized and disclosed. Inventories that have been pledged under loan agreements should be properly disclosed in the notes of the financial statements. Obtaining the results of confirmations would be the best procedure for obtaining this form of evidence. "Observing the company's physical inventory-taking procedures" is incorrect. This is an example of a test of control that verifies whether the company has adequate processes for managing and recording tangible assets. This audit technique may help the auditor assess the "existence" assertion. "Tracing the test counts during the physical inventory to the final inventory listing" is incorrect. This serves as an example of assessing the "completeness" assertion—in essence, that all inventory that should be presented in the financial statements has been so included. "Examining the inventory turnover ratio and determining whether it is in line with similar companies in the industry" is incorrect. This would be an example of an analytical procedure that might be performed for the purpose of identifying unusual trends or to verify the "valuation" assertion.

Which of the following procedures best addresses the adequacy of presentation and disclosure for inventory of a nonissuer? Obtaining confirmation of inventories that are pledged under loan agreements Observing the company's physical inventory-taking procedures Tracing the test counts during the physical inventory to the final inventory listing Examining the inventory turnover ratio and determining whether it is in line with similar companies in the industry

Inspect the shipping records documenting the merchandise and date sold to the debtors If an accounts receivable confirmation response is not received, the best procedure would be for the auditor to inspect shipping records documenting merchandise sold to the debtors. This procedure provides evidence that merchandise from a recorded sale was sent to the customer, substantiating the amount owed, which is the objective of the accounts receivable confirmation. An alternative procedure would be to review the cash receipts journal for the month following year-end to see that the customer paid the amount of the receivable.

Which of the following procedures would an auditor most likely perform for year-end accounts receivable confirmations when the auditor did not receive replies to second requests? Review the cash receipts journal for the month prior to the year-end Intensify the study of internal control concerning the revenue cycle Increase the assessed level of detection risk for the existence assertion Inspect the shipping records documenting the merchandise and date sold to the debtors

The accountant is independent with respect to the entity. An implicit representation is one that is implied rather than stated; an explicit representation is one that is fully and clearly expressed. If the accountant is not independent with respect to a client when issuing a compilation report, the accountant should specifically disclose the lack of independence in a final paragraph in the compilation report. Otherwise, the report contains the implicit representation that the accountant is independent. A review consists of inquiries and analytical procedures, not a compilation. The other two representations are made explicitly in the report: The financial statements have not been audited. The accountant does not express any assurance on the financial statements.

Which of the following representations does an accountant make implicitly when issuing the standard report for the compilation of a nonissuer's financial statements? The accountant is independent with respect to the entity. The financial statements have not been audited. A compilation consists principally of inquiries and analytical procedures. The accountant does not express any assurance on the financial statements.

Continuation of a reporting entity as a going concern is presumed as the basis for preparing financial statements unless and until the entity's liquidation becomes imminent. Only the statement, "Continuation of a reporting entity as a going concern is presumed as the basis for preparing financial statements unless and until the entity's liquidation becomes imminent," is correct. Substantial doubt about an entity's ability to continue as a going concern exists when relevant conditions and events, considered in the aggregate, indicate that it is probable that the entity will be unable to meet its obligations as they become due. No one can predict future events. There may be conditions or events that raise substantial doubt about the entity's ability to continue as a going concern; in those situations, financial statements should continue to be prepared under the going concern basis of accounting.

Which of the following statements regarding the going concern assumption is correct? Continuation of a reporting entity as a going concern is presumed as the basis for preparing financial statements unless and until the entity's liquidation becomes imminent. Substantial doubt about an entity's ability to continue as a going concern exists when relevant conditions and events indicate that it is possible the entity will be unable to meet its obligations as they become due. Management is responsible for predicting future conditions or events in assessing the likelihood that the entity will continue as a going concern. Financial statements should be prepared under the liquidation basis of accounting as soon as substantial doubt is raised.

Continuation of an entity as a going concern is assumed in financial reporting in the absence of significant information to the contrary. The statement "Continuation of an entity as a going concern is assumed in financial reporting in the absence of significant information to the contrary" is correct (AU-C 570.02). The auditor makes a decision based on relevant conditions and events that exist at, or have occurred prior to, the auditor's report.

Which of the following statements regarding the going concern assumption is correct? Continuation of an entity as a going concern is assumed in financial reporting in the absence of significant information to the contrary. The going concern concept reflects the entity's inability to meet its obligations. The auditor is responsible for predicting future conditions or events in assessing the likelihood that the entity will continue as a going concern. The auditor must apply audit procedures designed solely to identify conditions and events that indicate there could be substantial doubt about the entity's ability to continue as a going concern.

Information on control activities can be utilized to determine areas that need attention. The auditor should obtain an understanding of control activities relevant to the audit. An audit does not require an understanding of all control activities; the auditor would utilize information regarding the presence or absence of control activities (received from the understanding of other components of internal control) to determine whether it is necessary to devote attention to specific control activities.

Why should an auditor obtain an understanding of control activities relevant to an audit? A lack of control activities implies management improprieties Existence of control activities eliminates audit risk. Information on control activities can be utilized to determine areas that need attention. All of the answer choices are correct.


संबंधित स्टडी सेट्स

Real Estate Practices - Chapter 3: Agency and other Mandatory Disclosures

View Set

OLS-34200 Interview Strat Orgn - Midterm

View Set

Crozer Podiatry Manual 2nd Edition

View Set

The Fight Against Racial Injustice is Transatlantic

View Set

Developmental Concepts - OB Module 3

View Set

The CITI Good Clinical Practice Course for Clinical Trials Involving Drugs and Biologics Quiz

View Set

Entrepreneurship and Small Business Certification Exam

View Set